05 x05 Standard Costing & Variance Analysis

August 7, 2018 | Author: Ferjeanie Bernandino | Category: Cost Accounting, Labour Economics, Cost, Variance, Solution
Share Embed Donate


Short Description

Download 05 x05 Standard Costing & Variance Analysis...

Description

Standard Costs and Variance Analysis

STANDARD COSTS AND VARIANCE ANALYSIS

D. Practic Practical al capacit capacity y

THEORIES: Standard cost system 1. A primary purpose purpose of using using a standard cost cost system system is A. To make things easier easier for managers managers in the production facility. facility. B. To provide provide a distinct measure measure of cost cost control. control. C. To minimize minimize the cost cost per unit unit of production. production. D. b and c are are correc correctt

6. A company employing employing very very tight (high) standards standards in a standard standard cost system should expect that A. No incentive incentive bonus bonus will will be paid. B. Most variances variances will be be unfavorable. unfavorable. C. Employees Employees will be strongly strongly motivated motivated to attain the standard. standard. D. Costs will will be controlled better than if lower standards standards were used. 7. To measur measure e control controllab lable le product production ion inefficie inefficienci ncies, es, which of the following is the best basis for a company to use in establishing the standard hours allowed for the output of one unit of product? A. Average Average historical historical performance performance for the last several several years B. Engineering Engineering estimates estimates based on ideal performanc performance e C. Engine Engineering ering estimates estimates based on attainable attainable performance D. The The hours hours per unit unit that that woul would d be requi require red d for the pres presen entt workforce to satisfy expected demand over the long run

2. Which one of the following following statements statements is true concerning concerning standard standard costs? A. Standard Standard costs are estimates estimates of costs costs attainable attainable only under the most ideal conditions, but rarely practicable. B. Standard Standard costs are difficult to use with a process-costin process-costing g system. C. If properly properly used, standards standards can help motivate motivate employees. employees. D. Unfa Unfavo vora rabl ble e vari varian ance ces, s, mate materi rial al in amou amount nt,, shou should ld be inve invest stig igat ated, ed, but larg large e favor favorab able le vari varian ances ces need need not not be investigated.

8. Which of the following following statements statements about about the selection selection of standards standards is true? A. Ideal standards standards tend to extract extract higher performance performance levels levels since they give employees something to live up to. B. Curre Currentl ntly y attai attaina nabl ble e stand standar ards ds may may encou encoura rage ge opera operati ting ng inefficiencies. C. Curren Currently tly attain attainabl able e standar standards ds discou discourag rage e employe employees es from achieving their full performance potential. D. Ideal standards standards demand maximum maximum efficiency efficiency which may leave workers frustrated, thus causing a decline in performance.

3. Which of the followi following ng is a purpose purpose of standard costing? costing? A. Determine Determine “breakeven” “breakeven” producti production on level B. Con Control trol cos costs ts C. Eliminate Eliminate the need for subjective subjective decisions decisions by management management D. Allocate Allocate cost with with more accuracy accuracy 4. When When eval evalua uati ting ng the the oper operati ating ng perfo perform rman ance ce manag managem ement ent someti sometimes mes uses uses the differen difference ce between between expecte expected d and actual actual performance. This refers to: A. Mana Manage geme ment nt by Devi Deviat atio ion n C. Mana Manage geme ment nt by Obje Object ctiv ive e B. Manage Management ment by Contr Control ol D. Managem Management ent by Exceptio Exception n

Standard costs vs. budgeted costs 9. A difference difference between standard standard costs used used for cost control and the budgeted costs representing representing the same manufacturing manufacturing effort can exist because A. stan standa dard rd cost costs s must must be dete determ rmin ined ed afte afterr the the budg budget et is completed B. standa standard rd costs represent represent what costs costs should be while while budgeted costs represent expected actual costs C. budgeted budgeted costs are histor historica icall costs costs while while standa standard rd costs costs are

Standard setting 5. The best basis basis upon which which cost standards standards should should be set to measure controllable production inefficiencies is A. Engineering Engineering standards standards based on ideal performanc performance e B. Normal Normal capaci capacity ty C. Engine Engineering ering standards standards based on attainable attainable performance performance 203

Standard Costs and Variance Analysis

based on engineering studies D. budge budgeted ted cost costs s incl includ ude e some some “sla “slack ck” ” or “pad “paddi ding ng” ” whil while e standard costs do not

14.Standards that represent levels of operation that can be attained with reasonable effort are called: A. Theoretical standards C. Variable standards B. Ideal Ideal standa standards rds D. Nor Normal mal standar standards ds

Process costing 10.When standard costs are used in a process-costing system, how, if  at all, are equivalent units involved or used in the cost report at standard? A. Equivalent Equivalent units units are are not used. B. Equivalent Equivalent units are computed computed using a “special” “special” approach. approach. C. The actual equivalent equivalent units units are multiplied multiplied by the standard cost per unit. D. The standard standard equivalent equivalent units are multiplied multiplied by the actual cost per unit.

Variances Generic variances 15.Whe 15.When n perform performing ing input/ input/out output put varianc variance e analysi analysis s in standa standard rd costing, “standard hours allowed” is a means of measuring A. Standard Standard output output at standard standard hours C. Actual output at standard hours B. Stan Standa dard rd outp output ut at actu actual al hour hours s D. Actu Actual al outp output ut at actu actual al hours  Two way variances Volume variance 16.A 16.A company company uses uses a two-wa two-way y analysi analysis s for overhea overhead d variance variances: s: budget (controllable) and volume. The volume variance is based on the A. Total overhead overhead applicatio application n rate B. Volume Volume of total expenses expenses at various activit activity y levels C. Variable Variable overhead overhead application application rate rate D. Fixed overhead overhead applica application tion rate

Normal costing 11.The fixed overhead application rate is a function of a predetermined “normal” “normal” activity level. level. If standard hours allowed allowed for good output equal equal this this predeter predetermin mined ed activi activity ty level level for a given given period period,, the volume variance will be A. Ze ro ro B. Favo Favorab rable le C. Unfa Unfavo vora rable ble D. Either Either favorable favorable or unfavo unfavorabl rable, e, dependi depending ng on the budgeted budgeted overhead.

17.Assuming that the standard fixed overhead rate is based on full capacity, the cost of available but unused productive capacity is indicated by the: A. Facto Factory ry overhead overhead cost volume volume variance variance B. Direct Direct labor cost cost efficiency efficiency variance variance C. Direct Direct labor cost cost rate rate variance variance D. Factory Factory overhead cost controlla controllable ble variance variance

Types of standards 12.The absolute minimum cost possible under the best conceivable operating conditions is a description of which type of standard? A. Currently Currently attainab attainable le (expected) (expected) C. The or oret ic ical B. No Normal D. Practical 13.Standards, which are difficult to achieve due to reasons beyond the individual performing the task, are the result of firm using which of  the following methods to establish standards? A. Ide Ideal al Stan Standard dards s C. Practi Practical cal Standar Standards ds B. Lax Standards D. Employee Standards

18.In 18.In analyzi analyzing ng manufa manufactur cturing ing overhea overhead d varian variances ces,, the volume volume variance is the difference between the: A. Amount shown shown in the flexible flexible budget and the amount amount shown in the debit side of the overhead control account B. Predetermined Predetermined overhead overhead application application rate and the flexible flexible budget application rate times actual hours worked 204

Standard Costs and Variance Analysis

C. Budget allowance based on standard hours allowed for actual production for the period and the amount budgeted to be applied during the period D. Actual amount spent for overhead items during the period and the overhead amount applied to production during the period 19.The variance least significant for purposes of controlling costs is the: A. Material usage variance B. Variable overhead efficiency variance C. Fixed overhead spending variance D. Fixed overhead volume variance 20.The variance most useful in evaluating plant utilization is the: A. Variable overhead spending variance B. Fixed overhead spending variance. C. Variable overhead efficiency variance D. Fixed overhead volume variance Four way variances 21.The choice of production volume as a denominator for calculating its factory overhead rate A. Has no effect on the fixed factory overhead rate for applying costs to production B. Has an effect on the variable factory overhead rate for applying costs to production C. Has no effect on the fixed factory overhead budget variance D. Has no effect on the fixed factory overhead production volume variance

23.Which department is customarily held responsible unfavorable materials usage variance? A. Quality control C. Purchasing B. Engineering D. Production

for

an

Variance analysis 24.Which of the following should be least considered when deciding whether to investigate a variance? A. Whether the variance is favorable or unfavorable B. Significance of the variance C. Cost of investigating the variance D. Trend of the variances over time  Total materials variance 25.If the total materials variance (actual cost of materials used compared with the standard cost of the standard amount of  materials required) for a given operation is favorable, why must this variance be further evaluated as to price and usage? A. There is no need to further evaluate the total materials variance if it is favorable B. Generally accepted accounting principles require that all variances be analyzed in three stages C. All variances must appear in the annual report to equity owners for proper disclosure D. To allow management to evaluate the efficiency of the purchasing and production functions Labor variances 26.Which of the following unfavorable cost variances would be directly

Standard Costs and Variance Analysis

based on engineering studies D. budge budgeted ted cost costs s incl includ ude e some some “sla “slack ck” ” or “pad “paddi ding ng” ” whil while e standard costs do not

14.Standards that represent levels of operation that can be attained with reasonable effort are called: A. Theoretical standards C. Variable standards B. Ideal Ideal standa standards rds D. Nor Normal mal standar standards ds

Process costing 10.When standard costs are used in a process-costing system, how, if  at all, are equivalent units involved or used in the cost report at standard? A. Equivalent Equivalent units units are are not used. B. Equivalent Equivalent units are computed computed using a “special” “special” approach. approach. C. The actual equivalent equivalent units units are multiplied multiplied by the standard cost per unit. D. The standard standard equivalent equivalent units are multiplied multiplied by the actual cost per unit.

Variances Generic variances 15.Whe 15.When n perform performing ing input/ input/out output put varianc variance e analysi analysis s in standa standard rd costing, “standard hours allowed” is a means of measuring A. Standard Standard output output at standard standard hours C. Actual output at standard hours B. Stan Standa dard rd outp output ut at actu actual al hour hours s D. Actu Actual al outp output ut at actu actual al hours  Two way variances Volume variance 16.A 16.A company company uses uses a two-wa two-way y analysi analysis s for overhea overhead d variance variances: s: budget (controllable) and volume. The volume variance is based on the A. Total overhead overhead applicatio application n rate B. Volume Volume of total expenses expenses at various activit activity y levels C. Variable Variable overhead overhead application application rate rate D. Fixed overhead overhead applica application tion rate

Normal costing 11.The fixed overhead application rate is a function of a predetermined “normal” “normal” activity level. level. If standard hours allowed allowed for good output equal equal this this predeter predetermin mined ed activi activity ty level level for a given given period period,, the volume variance will be A. Ze ro ro B. Favo Favorab rable le C. Unfa Unfavo vora rable ble D. Either Either favorable favorable or unfavo unfavorabl rable, e, dependi depending ng on the budgeted budgeted overhead.

17.Assuming that the standard fixed overhead rate is based on full capacity, the cost of available but unused productive capacity is indicated by the: A. Facto Factory ry overhead overhead cost volume volume variance variance B. Direct Direct labor cost cost efficiency efficiency variance variance C. Direct Direct labor cost cost rate rate variance variance D. Factory Factory overhead cost controlla controllable ble variance variance

Types of standards 12.The absolute minimum cost possible under the best conceivable operating conditions is a description of which type of standard? A. Currently Currently attainab attainable le (expected) (expected) C. The or oret ic ical B. No Normal D. Practical 13.Standards, which are difficult to achieve due to reasons beyond the individual performing the task, are the result of firm using which of  the following methods to establish standards? A. Ide Ideal al Stan Standard dards s C. Practi Practical cal Standar Standards ds B. Lax Standards D. Employee Standards

18.In 18.In analyzi analyzing ng manufa manufactur cturing ing overhea overhead d varian variances ces,, the volume volume variance is the difference between the: A. Amount shown shown in the flexible flexible budget and the amount amount shown in the debit side of the overhead control account B. Predetermined Predetermined overhead overhead application application rate and the flexible flexible budget application rate times actual hours worked 204

Standard Costs and Variance Analysis

C. Budget allowance based on standard hours allowed for actual production for the period and the amount budgeted to be applied during the period D. Actual amount spent for overhead items during the period and the overhead amount applied to production during the period

23.Which department is customarily held responsible unfavorable materials usage variance? A. Quality control C. Purchasing B. Engineering D. Production

for

an

Variance analysis 24.Which of the following should be least considered when deciding whether to investigate a variance? A. Whether the variance is favorable or unfavorable B. Significance of the variance C. Cost of investigating the variance D. Trend of the variances over time

19.The variance least significant for purposes of controlling costs is the: A. Material usage variance B. Variable overhead efficiency variance C. Fixed overhead spending variance D. Fixed overhead volume variance 20.The variance most useful in evaluating plant utilization is the: A. Variable overhead spending variance B. Fixed overhead spending variance. C. Variable overhead efficiency variance D. Fixed overhead volume variance

 Total materials variance 25.If the total materials variance (actual cost of materials used compared with the standard cost of the standard amount of  materials required) for a given operation is favorable, why must this variance be further evaluated as to price and usage? A. There is no need to further evaluate the total materials variance if it is favorable B. Generally accepted accounting principles require that all variances be analyzed in three stages C. All variances must appear in the annual report to equity owners for proper disclosure D. To allow management to evaluate the efficiency of the purchasing and production functions

Four way variances 21.The choice of production volume as a denominator for calculating its factory overhead rate A. Has no effect on the fixed factory overhead rate for applying costs to production B. Has an effect on the variable factory overhead rate for applying costs to production C. Has no effect on the fixed factory overhead budget variance D. Has no effect on the fixed factory overhead production volume variance

Labor variances 26.Which of the following unfavorable cost variances would be directly affected by the relative position of a production process on a learning curve? A. Materials mix C. Labor rate B. Materials price D. Labor efficiency

22.The budgeted overhead costs for standard hours allowed and the overhead costs applied to product are the same amount A. for both variable and fixed overhead costs. B. only when standard hours allowed is less than normal capacity. C. for variable overhead costs. D. for fixed overhead costs.

27.Which of the following is the most probable reason with a company would experience an unfavorable labor rate variance and a favorable labor efficiency variance? A. The mix of workers assigned to the particular job was heavily

Responsibility for variances 205

Standard Costs and Variance Analysis

weighted toward the use of higherly paid, experienced individuals. B. The mix of workers assigned to the particular job was heavily weighted toward the use of new, relatively low paid, unskilled workers. C. Because of the productive schedule, workers from other production areas were assigned to assist in this particular process. D. Defective materials caused more labor to be used in order to produce a standard unit.  Two-way overhead variance 28.The budget for a given cost during a given period was P1,600,000.  The actual cost for the period was P1,440,000. Considering these facts, it can be said that the plant manager has done a better than expected job in controlling the cost if: A. The cost is variable and actual production was 90% of budgeted production B. The cost is variable and actual production equaled budgeted production C. The cost is variable and actual production was 80% of budgeted production D. The cost is discretionary fixed cost and actual production equaled budgeted production Budget variance 29.The budget variance for fixed factory overhead for the normalvolume, practical-capacity, and expected-activity levels would be the:

based on 5,000 gallon capacity. Therefore, the cost of storing 4,000 gallons was P1,000 more (P.25 x 4,000) in total than if you had stored 5,000 gallons of liquid in the tank. Which variance is being described? A. Variable-overhead efficiency variance B. Fixed-overhead spending variance C. Variable-overhead spending variance D. Fixed-overhead volume variance 31.Favorable fixed overhead volume variance occurs if: A. There is a favorable labor efficiency variance B. There is a favorable labor rate variance C. Production is less than planned D. Production is greater than planned 32.The unfavorable volume variance may be due to all but which of the following factors? A. Failure to maintain an even flow of work B. Machine breakdowns C. Unexpected increases in the cost of utilities D. Failure to obtain enough sales orders 33.How will a favorable volume variance affect net income under each of the following methods? Absorpt Variabl ion e A. Decrea No se eff  ect

Standard Costs and Variance Analysis

C. Budget allowance based on standard hours allowed for actual production for the period and the amount budgeted to be applied during the period D. Actual amount spent for overhead items during the period and the overhead amount applied to production during the period

23.Which department is customarily held responsible unfavorable materials usage variance? A. Quality control C. Purchasing B. Engineering D. Production

for

an

Variance analysis 24.Which of the following should be least considered when deciding whether to investigate a variance? A. Whether the variance is favorable or unfavorable B. Significance of the variance C. Cost of investigating the variance D. Trend of the variances over time

19.The variance least significant for purposes of controlling costs is the: A. Material usage variance B. Variable overhead efficiency variance C. Fixed overhead spending variance D. Fixed overhead volume variance 20.The variance most useful in evaluating plant utilization is the: A. Variable overhead spending variance B. Fixed overhead spending variance. C. Variable overhead efficiency variance D. Fixed overhead volume variance

 Total materials variance 25.If the total materials variance (actual cost of materials used compared with the standard cost of the standard amount of  materials required) for a given operation is favorable, why must this variance be further evaluated as to price and usage? A. There is no need to further evaluate the total materials variance if it is favorable B. Generally accepted accounting principles require that all variances be analyzed in three stages C. All variances must appear in the annual report to equity owners for proper disclosure D. To allow management to evaluate the efficiency of the purchasing and production functions

Four way variances 21.The choice of production volume as a denominator for calculating its factory overhead rate A. Has no effect on the fixed factory overhead rate for applying costs to production B. Has an effect on the variable factory overhead rate for applying costs to production C. Has no effect on the fixed factory overhead budget variance D. Has no effect on the fixed factory overhead production volume variance

Labor variances 26.Which of the following unfavorable cost variances would be directly affected by the relative position of a production process on a learning curve? A. Materials mix C. Labor rate B. Materials price D. Labor efficiency

22.The budgeted overhead costs for standard hours allowed and the overhead costs applied to product are the same amount A. for both variable and fixed overhead costs. B. only when standard hours allowed is less than normal capacity. C. for variable overhead costs. D. for fixed overhead costs.

27.Which of the following is the most probable reason with a company would experience an unfavorable labor rate variance and a favorable labor efficiency variance? A. The mix of workers assigned to the particular job was heavily

Responsibility for variances 205

Standard Costs and Variance Analysis

weighted toward the use of higherly paid, experienced individuals. B. The mix of workers assigned to the particular job was heavily weighted toward the use of new, relatively low paid, unskilled workers. C. Because of the productive schedule, workers from other production areas were assigned to assist in this particular process. D. Defective materials caused more labor to be used in order to produce a standard unit.

based on 5,000 gallon capacity. Therefore, the cost of storing 4,000 gallons was P1,000 more (P.25 x 4,000) in total than if you had stored 5,000 gallons of liquid in the tank. Which variance is being described? A. Variable-overhead efficiency variance B. Fixed-overhead spending variance C. Variable-overhead spending variance D. Fixed-overhead volume variance 31.Favorable fixed overhead volume variance occurs if: A. There is a favorable labor efficiency variance B. There is a favorable labor rate variance C. Production is less than planned D. Production is greater than planned

 Two-way overhead variance 28.The budget for a given cost during a given period was P1,600,000.  The actual cost for the period was P1,440,000. Considering these facts, it can be said that the plant manager has done a better than expected job in controlling the cost if: A. The cost is variable and actual production was 90% of budgeted production B. The cost is variable and actual production equaled budgeted production C. The cost is variable and actual production was 80% of budgeted production D. The cost is discretionary fixed cost and actual production equaled budgeted production

32.The unfavorable volume variance may be due to all but which of the following factors? A. Failure to maintain an even flow of work B. Machine breakdowns C. Unexpected increases in the cost of utilities D. Failure to obtain enough sales orders 33.How will a favorable volume variance affect net income under each of the following methods? Absorpt Variabl ion e A. Decrea No se eff  ect B. Decrea Increa se se C. Increas No e eff  ect D. Increas Decre e ase

Budget variance 29.The budget variance for fixed factory overhead for the normalvolume, practical-capacity, and expected-activity levels would be the: A. Same except for normal volume C. Same except for expected activity B. Same except for practical capacity D. Same for all three activity levels Volume variance 30.You have leased a 5,000-gallon storage tank for P5,000 per month.  You stored 4,000 gallons of liquid in the tan k during the month. The cost of storage was P1.25 per gallon, rather than P1.00 per gallon

34.Favorable volume variances may be harmful when: 206

Standard Costs and Variance Analysis

A. B. C. D.

Machine repairs cause work stoppages Supervisors fail to maintain an even flow of work Production in excess of normal capacity cannot be sold There are insufficient sales orders to keep the factory operating at normal capacity

 Three-way Overhead variance 35.During 2006, a department’s three-variance overhead standard costing system reported unfavorable spending and volume variances. The activity level selected for allocating overhead to the product was based on 80% of practical capacity. It 100% of  practical capacity had been selected instead, how would the reported unfavorable spending and volume variances be affected? Spending Volume Variance Variance A. Increased Unchanged B. Increased Increased C. Unchanged Increased D. Unchanged Unchanged

 The purchase prices of the raw materials used in the manufacture of this new chemical solution are as follows: Nyclyn P15.00 per kilogram Salex P21.00 per liter Protet P28.00 per kilogram  The total standard materials cost of 20 liters of the product is: A. P1,043.20 C. P 834.56 B. P1,304.00 D. P1,234.00 ii

.

El Andre Co. uses a standard costing system in connection with the manufacture of a line of T-shirts. Each unit of finished product contains 2.25 yards of direct material. However, a 25 percent direct material spoilage calculated on input quantities occurs during the manufacturing process. The cost of the direct materials is P150 per yard. The standard direct material cost per unit of finished product is A. P 253 C. P 450 B. P 422 D. P 405

iii

PROBLEMS: Standard setting Raw Materials i . Shampoo Company is a chemical manufacturer that supplies industrial users. The company plans to introduce a new chemical solution and needs to develop a standard product cost for this new solution.

. Each finished unit of Product EM contains 60 pounds of raw material. The manufacturing process must provide for a 20% waste allowance. The raw material can be purchased for P2.50 a pound under terms of 2/10, n/30. The company takes all cash discounts.  The standard direct material cost for each unit of EM is: A. P180.00 C. P187.50 B. P183.75 D. P176.40

iv

. The Vandana Company has a signature scarf for ladies that is very popular. Certain production and marketing data are indicated

Standard Costs and Variance Analysis

weighted toward the use of higherly paid, experienced individuals. B. The mix of workers assigned to the particular job was heavily weighted toward the use of new, relatively low paid, unskilled workers. C. Because of the productive schedule, workers from other production areas were assigned to assist in this particular process. D. Defective materials caused more labor to be used in order to produce a standard unit.

based on 5,000 gallon capacity. Therefore, the cost of storing 4,000 gallons was P1,000 more (P.25 x 4,000) in total than if you had stored 5,000 gallons of liquid in the tank. Which variance is being described? A. Variable-overhead efficiency variance B. Fixed-overhead spending variance C. Variable-overhead spending variance D. Fixed-overhead volume variance 31.Favorable fixed overhead volume variance occurs if: A. There is a favorable labor efficiency variance B. There is a favorable labor rate variance C. Production is less than planned D. Production is greater than planned

 Two-way overhead variance 28.The budget for a given cost during a given period was P1,600,000.  The actual cost for the period was P1,440,000. Considering these facts, it can be said that the plant manager has done a better than expected job in controlling the cost if: A. The cost is variable and actual production was 90% of budgeted production B. The cost is variable and actual production equaled budgeted production C. The cost is variable and actual production was 80% of budgeted production D. The cost is discretionary fixed cost and actual production equaled budgeted production

32.The unfavorable volume variance may be due to all but which of the following factors? A. Failure to maintain an even flow of work B. Machine breakdowns C. Unexpected increases in the cost of utilities D. Failure to obtain enough sales orders 33.How will a favorable volume variance affect net income under each of the following methods? Absorpt Variabl ion e A. Decrea No se eff  ect B. Decrea Increa se se C. Increas No e eff  ect D. Increas Decre e ase

Budget variance 29.The budget variance for fixed factory overhead for the normalvolume, practical-capacity, and expected-activity levels would be the: A. Same except for normal volume C. Same except for expected activity B. Same except for practical capacity D. Same for all three activity levels Volume variance 30.You have leased a 5,000-gallon storage tank for P5,000 per month.  You stored 4,000 gallons of liquid in the tan k during the month. The cost of storage was P1.25 per gallon, rather than P1.00 per gallon

34.Favorable volume variances may be harmful when: 206

Standard Costs and Variance Analysis

A. B. C. D.

Machine repairs cause work stoppages Supervisors fail to maintain an even flow of work Production in excess of normal capacity cannot be sold There are insufficient sales orders to keep the factory operating at normal capacity

 The purchase prices of the raw materials used in the manufacture of this new chemical solution are as follows: Nyclyn P15.00 per kilogram Salex P21.00 per liter Protet P28.00 per kilogram  The total standard materials cost of 20 liters of the product is: A. P1,043.20 C. P 834.56 B. P1,304.00 D. P1,234.00

 Three-way Overhead variance 35.During 2006, a department’s three-variance overhead standard costing system reported unfavorable spending and volume variances. The activity level selected for allocating overhead to the product was based on 80% of practical capacity. It 100% of  practical capacity had been selected instead, how would the reported unfavorable spending and volume variances be affected? Spending Volume Variance Variance A. Increased Unchanged B. Increased Increased C. Unchanged Increased D. Unchanged Unchanged

ii

.

El Andre Co. uses a standard costing system in connection with the manufacture of a line of T-shirts. Each unit of finished product contains 2.25 yards of direct material. However, a 25 percent direct material spoilage calculated on input quantities occurs during the manufacturing process. The cost of the direct materials is P150 per yard. The standard direct material cost per unit of finished product is A. P 253 C. P 450 B. P 422 D. P 405

iii

. Each finished unit of Product EM contains 60 pounds of raw material. The manufacturing process must provide for a 20% waste allowance. The raw material can be purchased for P2.50 a pound under terms of 2/10, n/30. The company takes all cash discounts.  The standard direct material cost for each unit of EM is: A. P180.00 C. P187.50 B. P183.75 D. P176.40

PROBLEMS: Standard setting Raw Materials i . Shampoo Company is a chemical manufacturer that supplies industrial users. The company plans to introduce a new chemical solution and needs to develop a standard product cost for this new solution.

iv

. The Vandana Company has a signature scarf for ladies that is very popular. Certain production and marketing data are indicated below: Cost per yard of cloth P40.00 Allowance for rejected scarf 5% of production  Yards of cloth needed per scarf 0.475 yard Airfreight from supplier P1.00/yard Motor freight to customers P0.90 /scarf   Purchase discounts from supplier 3% Sales discount to customers 2%  The allowance for rejected scarf is not part of the 0.475 yard of 

 The new chemical solution is made by combining a chemical compound (Nyclyn) and a solution (Salex), boiling the mixture; adding a second compound (Protet), and bottling the resulting solution in 20-liter containers. The initial mix, which is 20 liters in volume, consists of 24 kilograms of Nyclyn and 19.2 liters of Salex. A 20% reduction in volume occurs during the boiling process. The solution is then cooled slightly before 10 kilograms of Protet are added; the addition of Protet does not affect the total liquid volume.

207

Standard Costs and Variance Analysis

cloth per scarf. Rejects have no market value. Materials are used at the start of production. Calculate the standard cost of cloth per scarf that Vandana Company should use in its cost sheets. A. P19.85 C. P19.40 B. P20.00 D. P19.90 Direct labor v . Double M company is a chemical manufacturer that supplies various products to industrial users. The company plans to introduce a new chemical solution called Bysap, for which it needs to develop a standard product cost. The following labor information is available on the production of Bysap.  The product, which is bottled in 10-liter containers, is primarily a mixture of Byclyn, Salex, and Protet.  The finished product is highly unstable, and one 10-liter batch out of six is rejected at final inspection. Rejected batches have no commercial value and are thrown out. It takes a worker 35 minutes to process one 10-liter batch of  Bysap. Employees work on eight-hour a day, including one hour per day for rest breaks and cleanup. What is the standard labor time to produce one 10-liter batch of  Bysap? A. 35 minutes C. 48 minutes B. 40 minutes D. 45 minutes

Materials & Labor Questions Nos. 7 and 8 are based on the following: Supercold Company is a small producer of fruit-flavored frozen desserts. For many years, Supercold’s products have had strong regional sales on the basis of brand recognition. However, other companies have begun marketing similar products in the area, and price competition has become increasingly important. Haydee Mejia, the company’s controller, is planning to implement a standard costing system for Supercold and has gathered considerable information on production and material requirements for Supercold’s products. Haydee believes that the use of standard costing will allow Supercold to improve cost control and make better pricing decisions.







Supercold’s most popular products is strawberry sherbet. The sherbet is produced in 10-gallon batches, and each batch requires six quarts of  good strawberries. The fresh strawberries are sorted by hand before entering the production process. Because of imperfections in the strawberries and normal spoilage, one quart of berries is discarded for every four quarts of acceptable berries. Three minutes is the standard direct labor time for sorting that is required to obtain one quart of  acceptable berries. The acceptable berries are then blended with the other ingredients; blending requires 12 minutes of direct labor time per batch. After blending, the sherbet is package in quart containers. Haydee has gathered the following information from Rizza Alano, Supercold’s cost accountant.

vi

. The following direct labor information pertains to the manufacture of Part J35: Number of hours required to make a part 2 5 DLH

Supercold purchases strawberries at a cost of P8.00 per quart. other ingredients cost a total of P4.50 per gallon.

All

Standard Costs and Variance Analysis

A. B. C. D.

Machine repairs cause work stoppages Supervisors fail to maintain an even flow of work Production in excess of normal capacity cannot be sold There are insufficient sales orders to keep the factory operating at normal capacity

 The purchase prices of the raw materials used in the manufacture of this new chemical solution are as follows: Nyclyn P15.00 per kilogram Salex P21.00 per liter Protet P28.00 per kilogram  The total standard materials cost of 20 liters of the product is: A. P1,043.20 C. P 834.56 B. P1,304.00 D. P1,234.00

 Three-way Overhead variance 35.During 2006, a department’s three-variance overhead standard costing system reported unfavorable spending and volume variances. The activity level selected for allocating overhead to the product was based on 80% of practical capacity. It 100% of  practical capacity had been selected instead, how would the reported unfavorable spending and volume variances be affected? Spending Volume Variance Variance A. Increased Unchanged B. Increased Increased C. Unchanged Increased D. Unchanged Unchanged

ii

.

El Andre Co. uses a standard costing system in connection with the manufacture of a line of T-shirts. Each unit of finished product contains 2.25 yards of direct material. However, a 25 percent direct material spoilage calculated on input quantities occurs during the manufacturing process. The cost of the direct materials is P150 per yard. The standard direct material cost per unit of finished product is A. P 253 C. P 450 B. P 422 D. P 405

iii

. Each finished unit of Product EM contains 60 pounds of raw material. The manufacturing process must provide for a 20% waste allowance. The raw material can be purchased for P2.50 a pound under terms of 2/10, n/30. The company takes all cash discounts.  The standard direct material cost for each unit of EM is: A. P180.00 C. P187.50 B. P183.75 D. P176.40

PROBLEMS: Standard setting Raw Materials i . Shampoo Company is a chemical manufacturer that supplies industrial users. The company plans to introduce a new chemical solution and needs to develop a standard product cost for this new solution.

iv

. The Vandana Company has a signature scarf for ladies that is very popular. Certain production and marketing data are indicated below: Cost per yard of cloth P40.00 Allowance for rejected scarf 5% of production  Yards of cloth needed per scarf 0.475 yard Airfreight from supplier P1.00/yard Motor freight to customers P0.90 /scarf   Purchase discounts from supplier 3% Sales discount to customers 2%  The allowance for rejected scarf is not part of the 0.475 yard of 

 The new chemical solution is made by combining a chemical compound (Nyclyn) and a solution (Salex), boiling the mixture; adding a second compound (Protet), and bottling the resulting solution in 20-liter containers. The initial mix, which is 20 liters in volume, consists of 24 kilograms of Nyclyn and 19.2 liters of Salex. A 20% reduction in volume occurs during the boiling process. The solution is then cooled slightly before 10 kilograms of Protet are added; the addition of Protet does not affect the total liquid volume.

207

Standard Costs and Variance Analysis

cloth per scarf. Rejects have no market value. Materials are used at the start of production. Calculate the standard cost of cloth per scarf that Vandana Company should use in its cost sheets. A. P19.85 C. P19.40 B. P20.00 D. P19.90

Materials & Labor Questions Nos. 7 and 8 are based on the following: Supercold Company is a small producer of fruit-flavored frozen desserts. For many years, Supercold’s products have had strong regional sales on the basis of brand recognition. However, other companies have begun marketing similar products in the area, and price competition has become increasingly important. Haydee Mejia, the company’s controller, is planning to implement a standard costing system for Supercold and has gathered considerable information on production and material requirements for Supercold’s products. Haydee believes that the use of standard costing will allow Supercold to improve cost control and make better pricing decisions.

Direct labor v . Double M company is a chemical manufacturer that supplies various products to industrial users. The company plans to introduce a new chemical solution called Bysap, for which it needs to develop a standard product cost. The following labor information is available on the production of Bysap.  The product, which is bottled in 10-liter containers, is primarily a mixture of Byclyn, Salex, and Protet.  The finished product is highly unstable, and one 10-liter batch out of six is rejected at final inspection. Rejected batches have no commercial value and are thrown out. It takes a worker 35 minutes to process one 10-liter batch of  Bysap. Employees work on eight-hour a day, including one hour per day for rest breaks and cleanup. What is the standard labor time to produce one 10-liter batch of  Bysap? A. 35 minutes C. 48 minutes B. 40 minutes D. 45 minutes •

Supercold’s most popular products is strawberry sherbet. The sherbet is produced in 10-gallon batches, and each batch requires six quarts of  good strawberries. The fresh strawberries are sorted by hand before entering the production process. Because of imperfections in the strawberries and normal spoilage, one quart of berries is discarded for every four quarts of acceptable berries. Three minutes is the standard direct labor time for sorting that is required to obtain one quart of  acceptable berries. The acceptable berries are then blended with the other ingredients; blending requires 12 minutes of direct labor time per batch. After blending, the sherbet is package in quart containers. Haydee has gathered the following information from Rizza Alano, Supercold’s cost accountant.





vi

. The following direct labor information pertains to the manufacture of Part J35: Number of hours required to make a part 2.5 DLH Number of Direct workers 75 Number of total productive hours per week 3000 Weekly wages per worker P1,000 Laborers’ fringe benefits treated as direct labor costs 25% of  wages What is the standard direct labor cost per unit of Part J35? A. P62.500 C. P41.670 B. P78.125 D. P84.125

Supercold purchases strawberries at a cost of P8.00 per quart. All other ingredients cost a total of P4.50 per gallon. Direct labor is paid at the rate of P50 per hour.  The total cost of material an d labor require d to package the sherbet is P3.80 per quart. Rizza Alano has a friend who owns a berry farm that has been losing money in recent years. Because of good crops, there has been an oversupply of strawberries, and prices have dropped to P5.00 per quart. Rizza has arranged for Supercold to purchase strawberries form her friend and hopes that P8.00 per quart will help her friend’s farm 208

Standard Costs and Variance Analysis

become profitable again. xii

. Sheridan Company has a standard of 15 parts of component BB costing P1.50 each. Sheridan purchased 14,910 units of  component BB for P22,145. Sheridan generated a P220 favorable price variance and a P3,735 favorable quantity variance. If there were no changes in the component inventory, how many units of  finished product were produced? A. 994 units. C. 1,000 units B. 1,090 units. D. 1,160 units

vii

. The standard materials cost per 10-gallon batch of strawberry sherbet is: A. P 85.00 C. P101.00 B. P 60.00 D. P105.00

viii

. The standard direct labor cost per 10-gallon batch of sherbet is: A. P50.00 C. P25.00 B. P28.75 D. P15.00

xiii

Materials variance ix . Under a standard cost system, the materials quantity variance was recorded at P1,970 unfavorable, the materials price variance was recorded at P3,740 favorable, and the Goods in Process was debited for P51,690. Ninety-six thousand units were completed. What was the per unit price of the actual materials used? A. P0.52 each C. P0.54 eac B. P0.53 each D. P0.51 each

. The standard usage for raw materials is 5 pounds at P40.00 per pound. Cave Company spent P131,200 in purchasing 3,200 pounds. Cave used 3,150 pounds to produce 600 units of finished product. The material quantity variance is: A. P6,000 unfavorable C. P3,200 unfavorable B. P5,200 unfavorable D. P2,000 unfavorable

xiv

. The Bohol Company uses standard costing. The following data are available for October: Actual quantity of direct materials used 23,500 pounds Standard price of direct materials P2 per pound Material quantity variance P1,000 U  The standard quantity of materials allowed f or October production is: A. 23,000 lbs C. 24,000 lbs B. 24,500 lbs D. 25,000 lbs

x

. Blake Company has a standard price of P5.50 per pound for materials. July’s results showed an unfavorable material price variance of P44 and a favorable quantity variance of P209. If 1,066 pounds were used in production, what was the standard quantity allowed for materials? A . 1, 104 C. 1 ,0 66 B. 1,074 D. 1,100 xv

xi

. Elite Company uses a standard costing system in the manufacture

. Information on Dulce’s direct material costs for May is as follows: Actual quantity of direct materials purchased and used30,000

Standard Costs and Variance Analysis

cloth per scarf. Rejects have no market value. Materials are used at the start of production. Calculate the standard cost of cloth per scarf that Vandana Company should use in its cost sheets. A. P19.85 C. P19.40 B. P20.00 D. P19.90

Materials & Labor Questions Nos. 7 and 8 are based on the following: Supercold Company is a small producer of fruit-flavored frozen desserts. For many years, Supercold’s products have had strong regional sales on the basis of brand recognition. However, other companies have begun marketing similar products in the area, and price competition has become increasingly important. Haydee Mejia, the company’s controller, is planning to implement a standard costing system for Supercold and has gathered considerable information on production and material requirements for Supercold’s products. Haydee believes that the use of standard costing will allow Supercold to improve cost control and make better pricing decisions.

Direct labor v . Double M company is a chemical manufacturer that supplies various products to industrial users. The company plans to introduce a new chemical solution called Bysap, for which it needs to develop a standard product cost. The following labor information is available on the production of Bysap.  The product, which is bottled in 10-liter containers, is primarily a mixture of Byclyn, Salex, and Protet.  The finished product is highly unstable, and one 10-liter batch out of six is rejected at final inspection. Rejected batches have no commercial value and are thrown out. It takes a worker 35 minutes to process one 10-liter batch of  Bysap. Employees work on eight-hour a day, including one hour per day for rest breaks and cleanup. What is the standard labor time to produce one 10-liter batch of  Bysap? A. 35 minutes C. 48 minutes B. 40 minutes D. 45 minutes •

Supercold’s most popular products is strawberry sherbet. The sherbet is produced in 10-gallon batches, and each batch requires six quarts of  good strawberries. The fresh strawberries are sorted by hand before entering the production process. Because of imperfections in the strawberries and normal spoilage, one quart of berries is discarded for every four quarts of acceptable berries. Three minutes is the standard direct labor time for sorting that is required to obtain one quart of  acceptable berries. The acceptable berries are then blended with the other ingredients; blending requires 12 minutes of direct labor time per batch. After blending, the sherbet is package in quart containers. Haydee has gathered the following information from Rizza Alano, Supercold’s cost accountant.





vi

. The following direct labor information pertains to the manufacture of Part J35: Number of hours required to make a part 2.5 DLH Number of Direct workers 75 Number of total productive hours per week 3000 Weekly wages per worker P1,000 Laborers’ fringe benefits treated as direct labor costs 25% of  wages What is the standard direct labor cost per unit of Part J35? A. P62.500 C. P41.670 B. P78.125 D. P84.125

Supercold purchases strawberries at a cost of P8.00 per quart. All other ingredients cost a total of P4.50 per gallon. Direct labor is paid at the rate of P50 per hour.  The total cost of material an d labor require d to package the sherbet is P3.80 per quart. Rizza Alano has a friend who owns a berry farm that has been losing money in recent years. Because of good crops, there has been an oversupply of strawberries, and prices have dropped to P5.00 per quart. Rizza has arranged for Supercold to purchase strawberries form her friend and hopes that P8.00 per quart will help her friend’s farm 208

Standard Costs and Variance Analysis

become profitable again. xii

. Sheridan Company has a standard of 15 parts of component BB costing P1.50 each. Sheridan purchased 14,910 units of  component BB for P22,145. Sheridan generated a P220 favorable price variance and a P3,735 favorable quantity variance. If there were no changes in the component inventory, how many units of  finished product were produced? A. 994 units. C. 1,000 units B. 1,090 units. D. 1,160 units

vii

. The standard materials cost per 10-gallon batch of strawberry sherbet is: A. P 85.00 C. P101.00 B. P 60.00 D. P105.00

viii

. The standard direct labor cost per 10-gallon batch of sherbet is: A. P50.00 C. P25.00 B. P28.75 D. P15.00

xiii

. The standard usage for raw materials is 5 pounds at P40.00 per pound. Cave Company spent P131,200 in purchasing 3,200 pounds. Cave used 3,150 pounds to produce 600 units of finished product. The material quantity variance is: A. P6,000 unfavorable C. P3,200 unfavorable B. P5,200 unfavorable D. P2,000 unfavorable

Materials variance ix . Under a standard cost system, the materials quantity variance was recorded at P1,970 unfavorable, the materials price variance was recorded at P3,740 favorable, and the Goods in Process was debited for P51,690. Ninety-six thousand units were completed. What was the per unit price of the actual materials used? A. P0.52 each C. P0.54 eac B. P0.53 each D. P0.51 each

xiv

. The Bohol Company uses standard costing. The following data are available for October: Actual quantity of direct materials used 23,500 pounds Standard price of direct materials P2 per pound Material quantity variance P1,000 U  The standard quantity of materials allowed f or October production is: A. 23,000 lbs C. 24,000 lbs B. 24,500 lbs D. 25,000 lbs

x

. Blake Company has a standard price of P5.50 per pound for materials. July’s results showed an unfavorable material price variance of P44 and a favorable quantity variance of P209. If 1,066 pounds were used in production, what was the standard quantity allowed for materials? A . 1, 104 C. 1 ,0 66 B. 1,074 D. 1,100 xv

. Information on Dulce’s direct material costs for May is as follows: Actual quantity of direct materials purchased and used30,000 lbs. Actual cost of direct materials P84,000 U nf av or ab le d ir ec t m at er ia ls u sa ge v ar ia nc e P 3 ,000 Standard quantity of direct materials allowed for May production 29,000 lbs For the month of May, Dulce’s direct materials price variance was: A. P2,800 favorable C. P2,800 unfavorable B. P6,000 unfavorable D. P6,000 favorable

xi

. Elite Company uses a standard costing system in the manufacture of its single product. The 35,000 units of raw material in inventory were purchased for P105,000, and two units of raw material are required to produce one unit of final product. In November, the company produced 12,000 units of product. The standard allowed for material was P60,000, and there was an unfavorable quantity variance of P2,500. The materials price variance for the units used in November was A. P 2,500 U C. P12,500 U B. P11,000 U D. P 3,500 F 209

Standard Costs and Variance Analysis xvi

. Information on Katrina Company’s direct material costs is as follows: Standard unit price P 3.60 Actual quantity purchased 1,600 Standard quantity allowed for actual production 1,450 Mat erials pu rch ase price variance – favor able P 240 What was the actual purchase price per unit, rounded to the nearest centavos? A. P3.06 C. P3.11 B . P3 .4 5 D. P3.75

xvii

. Palmas Company, which has a standard cost system, had 500 units of raw material X in its inventory at June 1, purchased in May for P1.20 per unit and carried at a standard cost of P1.00. The following information pertains to raw material X for the month of June: Actual number of units purchased 1,400 Actual number of units used 1,500 Standard number of units allowed for actual production 1,300 Standard cost per unit P1.00 Actual cost per unit P1.10  The unfavorable materials purchase price variance for raw material X for June was: A. P 0 C. P140 B . P1 30 D. P 15 0

xviii

. During March, Lumban Company’s direct material costs for the manufacture of product T were as follows: Actual unit purchase price P6.50 S ta nd ar d qu an ti ty a ll owe d fo r a ct ua l pr od uc ti on 2, 10 0

 Type R houses, using 22,000 units of material cost of P26,400. Razonable’s material price variance for April is: A. P1,000 favorable C. P1,100 favorable B. P1,400 unfavorable D. P2,500 unfavorable xx

. Samson Candle Co. manufactures candles in various shapes, sizes, colors, and scents. Depending on the orders received, not all candles require the same amount of color, dye, or scent materials.  Yields also vary, depending upon the usage of beeswax or synthetic wax. Standard ingredients for 1,000 pounds of candles are: Input: Standard Mix Standard Cost per Pound Beeswax 200 lbs. 1.00 Synthetic wax 840 lbs. 0.20 Colors 7 lbs. 2.00 Scents 3 lbs. 6.00  Totals 1,050 lbs. 9.20 Standard 1,000 lbs. output Price variances are charged off at the time of purchase. During  January, the company was busy manufacturing red candles for Valentine’s Day. Actual production then was: Input: In Pounds Beeswax 4,100 Synthetic wax 13,800 Colors 2,200 Scents 60  Total 20,160 Actual output 18 500

Standard Costs and Variance Analysis

become profitable again. xii

. Sheridan Company has a standard of 15 parts of component BB costing P1.50 each. Sheridan purchased 14,910 units of  component BB for P22,145. Sheridan generated a P220 favorable price variance and a P3,735 favorable quantity variance. If there were no changes in the component inventory, how many units of  finished product were produced? A. 994 units. C. 1,000 units B. 1,090 units. D. 1,160 units

vii

. The standard materials cost per 10-gallon batch of strawberry sherbet is: A. P 85.00 C. P101.00 B. P 60.00 D. P105.00

viii

. The standard direct labor cost per 10-gallon batch of sherbet is: A. P50.00 C. P25.00 B. P28.75 D. P15.00

xiii

. The standard usage for raw materials is 5 pounds at P40.00 per pound. Cave Company spent P131,200 in purchasing 3,200 pounds. Cave used 3,150 pounds to produce 600 units of finished product. The material quantity variance is: A. P6,000 unfavorable C. P3,200 unfavorable B. P5,200 unfavorable D. P2,000 unfavorable

Materials variance ix . Under a standard cost system, the materials quantity variance was recorded at P1,970 unfavorable, the materials price variance was recorded at P3,740 favorable, and the Goods in Process was debited for P51,690. Ninety-six thousand units were completed. What was the per unit price of the actual materials used? A. P0.52 each C. P0.54 eac B. P0.53 each D. P0.51 each

xiv

. The Bohol Company uses standard costing. The following data are available for October: Actual quantity of direct materials used 23,500 pounds Standard price of direct materials P2 per pound Material quantity variance P1,000 U  The standard quantity of materials allowed f or October production is: A. 23,000 lbs C. 24,000 lbs B. 24,500 lbs D. 25,000 lbs

x

. Blake Company has a standard price of P5.50 per pound for materials. July’s results showed an unfavorable material price variance of P44 and a favorable quantity variance of P209. If 1,066 pounds were used in production, what was the standard quantity allowed for materials? A . 1, 104 C. 1 ,0 66 B. 1,074 D. 1,100 xv

. Information on Dulce’s direct material costs for May is as follows: Actual quantity of direct materials purchased and used30,000 lbs. Actual cost of direct materials P84,000 U nf av or ab le d ir ec t m at er ia ls u sa ge v ar ia nc e P 3 ,000 Standard quantity of direct materials allowed for May production 29,000 lbs For the month of May, Dulce’s direct materials price variance was: A. P2,800 favorable C. P2,800 unfavorable B. P6,000 unfavorable D. P6,000 favorable

xi

. Elite Company uses a standard costing system in the manufacture of its single product. The 35,000 units of raw material in inventory were purchased for P105,000, and two units of raw material are required to produce one unit of final product. In November, the company produced 12,000 units of product. The standard allowed for material was P60,000, and there was an unfavorable quantity variance of P2,500. The materials price variance for the units used in November was A. P 2,500 U C. P12,500 U B. P11,000 U D. P 3,500 F 209

Standard Costs and Variance Analysis xvi

. Information on Katrina Company’s direct material costs is as follows: Standard unit price P 3.60 Actual quantity purchased 1,600 Standard quantity allowed for actual production 1,450 Mat erials pu rch ase price variance – favor able P 240 What was the actual purchase price per unit, rounded to the nearest centavos? A. P3.06 C. P3.11 B . P3 .4 5 D. P3.75

 Type R houses, using 22,000 units of material cost of P26,400. Razonable’s material price variance for April is: A. P1,000 favorable C. P1,100 favorable B. P1,400 unfavorable D. P2,500 unfavorable xx

. Samson Candle Co. manufactures candles in various shapes, sizes, colors, and scents. Depending on the orders received, not all candles require the same amount of color, dye, or scent materials.  Yields also vary, depending upon the usage of beeswax or synthetic wax. Standard ingredients for 1,000 pounds of candles are: Input: Standard Mix Standard Cost per Pound Beeswax 200 lbs. 1.00 Synthetic wax 840 lbs. 0.20 Colors 7 lbs. 2.00 Scents 3 lbs. 6.00  Totals 1,050 lbs. 9.20 Standard 1,000 lbs. output Price variances are charged off at the time of purchase. During  January, the company was busy manufacturing red candles for Valentine’s Day. Actual production then was: Input: In Pounds Beeswax 4,100 Synthetic wax 13,800 Colors 2,200 Scents 60  Total 20,160 Actual output 18,500  The material yield variance is: A. P 280 unfavorable C. P 280 favorable B. P3,989 unfavorable D. P3,989 favorable

xvii

. Palmas Company, which has a standard cost system, had 500 units of raw material X in its inventory at June 1, purchased in May for P1.20 per unit and carried at a standard cost of P1.00. The following information pertains to raw material X for the month of June: Actual number of units purchased 1,400 Actual number of units used 1,500 Standard number of units allowed for actual production 1,300 Standard cost per unit P1.00 Actual cost per unit P1.10  The unfavorable materials purchase price variance for raw material X for June was: A. P 0 C. P140 B . P1 30 D. P 15 0

xviii

. During March, Lumban Company’s direct material costs for the manufacture of product T were as follows: Actual unit purchase price P6.50 S ta nd ar d qu an ti ty a ll owe d fo r a ct ua l pr od uc ti on 2, 10 0 Quantity purchased and used for actual production 2,300 Standard unit price P6.25 Lumban’s material usage variance for March was: A. P1,250 unfavorable C. P1,250 favorable B. P1,300 unfavorable D. P1,300 favorable

Labor variance xxi . The flexible budget for the month of May 2007 was for 9,000 units with direct material at P15 per unit. Direct labor was budgeted at 45 minutes per unit for a total of P81,000. Actual output for the month was 8,500 units with P127,500 in direct material and

xix

. Razonable Company installs shingle roofs on houses. The standard material cost for a Type R house is P1,250, based on 1,000 units at a cost of P1.25 each. During April, Razonable installed roofs on 20 210

Standard Costs and Variance Analysis

P77,775 in direct labor expense. Direct labor hours of 6,375 were actually worked during the month. Variance analysis of the performance for the month of May would show a(n): A. Favorable material quantity variance of P7,500. B. Unfavorable direct labor efficiency variance of P1,275. C. Unfavorable material quantity variance of P7,500. D. Unfavorable direct labor rate variance of P1,275. xxii

. The standard hourly rate was P4.10. Standard hours for the level of  production are 4,000. The actual rate was P4.27. The labor rate variance was P654.50, unfavorable. What were the actual labor hours? A . 3, 700 C. 3 ,8 50 B. 4,150 D. 4,000

xxiii

. Clean Harry Corp. uses two different types of labor to manufacture its product. The types of labor, Mixing and Finishing, have the following standards: Labor Type Standard Mix Std Hourly Standard Rate Cost Mixing 500 hours P10 P5,000 Finishing 250 hours P5 P1,250  Yield: 4,000 units During January, the following actual production information was provided: Labor Type Actual Mix Mixing 4,500 hours Finishing 3,000 hours  Yield: 36,000 units

xxv

. Hingis had a P750 unfavorable direct labor rate variance and an P800 favorable efficiency variance. Hingis paid P7,150 for 800 hours of labor. What was the standard direct labor wage rate? A . P8. 94 C. P 8. 00 B. P7.94 D. P7.80

xxvi

.Powerless Company’s operations for April disclosed the following data relating to direct labor: Actual cost P10,000 Rate variance 1,000 favorable Efficiency variance 1,500 unfavorable Standard cost P 9,500 Actual direct labor hours for April amounted to 2,000. Powerless’ standard direct labor rate per hour in April was: A . P5. 50 C. P 5. 00 B. P4.75 D. P4.50

xxvii

. Lion Company’s direct labor costs for the month of January were as follows: Actual direct labor hours 20,000 Standard direct labor hours 21,000 Direct labor rate variance – Unfav. P 3,000  Total payroll P126,000 What was Lion’s direct labor efficiency variance? A. P6,000 favorable C. P6,150 favorable B. P6,300 favorable D. P6,450 favorable

Standard Costs and Variance Analysis xvi

. Information on Katrina Company’s direct material costs is as follows: Standard unit price P 3.60 Actual quantity purchased 1,600 Standard quantity allowed for actual production 1,450 Mat erials pu rch ase price variance – favor able P 240 What was the actual purchase price per unit, rounded to the nearest centavos? A. P3.06 C. P3.11 B . P3 .4 5 D. P3.75

 Type R houses, using 22,000 units of material cost of P26,400. Razonable’s material price variance for April is: A. P1,000 favorable C. P1,100 favorable B. P1,400 unfavorable D. P2,500 unfavorable xx

. Samson Candle Co. manufactures candles in various shapes, sizes, colors, and scents. Depending on the orders received, not all candles require the same amount of color, dye, or scent materials.  Yields also vary, depending upon the usage of beeswax or synthetic wax. Standard ingredients for 1,000 pounds of candles are: Input: Standard Mix Standard Cost per Pound Beeswax 200 lbs. 1.00 Synthetic wax 840 lbs. 0.20 Colors 7 lbs. 2.00 Scents 3 lbs. 6.00  Totals 1,050 lbs. 9.20 Standard 1,000 lbs. output Price variances are charged off at the time of purchase. During  January, the company was busy manufacturing red candles for Valentine’s Day. Actual production then was: Input: In Pounds Beeswax 4,100 Synthetic wax 13,800 Colors 2,200 Scents 60  Total 20,160 Actual output 18,500  The material yield variance is: A. P 280 unfavorable C. P 280 favorable B. P3,989 unfavorable D. P3,989 favorable

xvii

. Palmas Company, which has a standard cost system, had 500 units of raw material X in its inventory at June 1, purchased in May for P1.20 per unit and carried at a standard cost of P1.00. The following information pertains to raw material X for the month of June: Actual number of units purchased 1,400 Actual number of units used 1,500 Standard number of units allowed for actual production 1,300 Standard cost per unit P1.00 Actual cost per unit P1.10  The unfavorable materials purchase price variance for raw material X for June was: A. P 0 C. P140 B . P1 30 D. P 15 0

xviii

. During March, Lumban Company’s direct material costs for the manufacture of product T were as follows: Actual unit purchase price P6.50 S ta nd ar d qu an ti ty a ll owe d fo r a ct ua l pr od uc ti on 2, 10 0 Quantity purchased and used for actual production 2,300 Standard unit price P6.25 Lumban’s material usage variance for March was: A. P1,250 unfavorable C. P1,250 favorable B. P1,300 unfavorable D. P1,300 favorable

Labor variance xxi . The flexible budget for the month of May 2007 was for 9,000 units with direct material at P15 per unit. Direct labor was budgeted at 45 minutes per unit for a total of P81,000. Actual output for the month was 8,500 units with P127,500 in direct material and

xix

. Razonable Company installs shingle roofs on houses. The standard material cost for a Type R house is P1,250, based on 1,000 units at a cost of P1.25 each. During April, Razonable installed roofs on 20 210

Standard Costs and Variance Analysis xxv

P77,775 in direct labor expense. Direct labor hours of 6,375 were actually worked during the month. Variance analysis of the performance for the month of May would show a(n): A. Favorable material quantity variance of P7,500. B. Unfavorable direct labor efficiency variance of P1,275. C. Unfavorable material quantity variance of P7,500. D. Unfavorable direct labor rate variance of P1,275.

. Hingis had a P750 unfavorable direct labor rate variance and an P800 favorable efficiency variance. Hingis paid P7,150 for 800 hours of labor. What was the standard direct labor wage rate? A . P8. 94 C. P 8. 00 B. P7.94 D. P7.80

xxvi

.Powerless Company’s operations for April disclosed the following data relating to direct labor: Actual cost P10,000 Rate variance 1,000 favorable Efficiency variance 1,500 unfavorable Standard cost P 9,500 Actual direct labor hours for April amounted to 2,000. Powerless’ standard direct labor rate per hour in April was: A . P5. 50 C. P 5. 00 B. P4.75 D. P4.50

xxii

. The standard hourly rate was P4.10. Standard hours for the level of  production are 4,000. The actual rate was P4.27. The labor rate variance was P654.50, unfavorable. What were the actual labor hours? A . 3, 700 C. 3 ,8 50 B. 4,150 D. 4,000

xxiii

. Clean Harry Corp. uses two different types of labor to manufacture its product. The types of labor, Mixing and Finishing, have the following standards: Labor Type Standard Mix Std Hourly Standard Rate Cost Mixing 500 hours P10 P5,000 Finishing 250 hours P5 P1,250  Yield: 4,000 units During January, the following actual production information was provided: Labor Type Actual Mix Mixing 4,500 hours Finishing 3,000 hours  Yield: 36,000 units What is the labor mix variance? A. P2,500 F C. P2,500 U B. P5,000 F D. P5,000 F

xxvii

. Lion Company’s direct labor costs for the month of January were as follows: Actual direct labor hours 20,000 Standard direct labor hours 21,000 Direct labor rate variance – Unfav. P 3,000  Total payroll P126,000 What was Lion’s direct labor efficiency variance? A. P6,000 favorable C. P6,150 favorable B. P6,300 favorable D. P6,450 favorable

xxviii

. Using the information given below, determine efficiency variance: Labor price per hour Standard labor price per gallon of output at 20 gal./hr Standard labor cost of 8,440 gallons of actual output Actual total inputs(410 hours at P21/hr) A. P 410 unfavorable C. P 240 favorable B. P 170 unfavorable D. P 410 favorable

xxiv

.How much labor yield variances should be reported? A. P6,250 U C. P5,250 F B. P6,250 F D. P5,250 U

the

labor

P 20 P 1 P8,440 P8,610

211

Standard Costs and Variance Analysis xxix

.Simbad Company’s operations for the month just ended originally set up a 60,000 direct labor hour level, with budgeted direct labor of P960,000 and budgeted variable overhead of P240,000. The actual results revealed that direct labor incurred amounted to P1,148,000 and that the unfavorable variable overhead variance was P40,000. Labor trouble caused an unfavorable labor efficiency variance of P120,000, and new employees hired at higher rates resulted in an actual average wage rate of P16.40 per hour. The total number of standard direct labor hours allowed for the actual units produced is A. P52,500 C. P62,500 B. P77,500 D. P70,000

Questions 30 and 31 are based on the following information. Information on Goodeve Company’s direct labor costs are presented below: Standard direct labor hours 30,000 Actual direct labor hours 29,000 Direct labor efficiency variance Favorable P 4,000 Direct labor rate variance Favorable P 5,800  Total payroll P110,200 xxx

. What was Goodeve’s standard direct labor rate? A. P3.54 C. P3.80 B . P4 .0 0 D. P5.80

xxxi

.What was Goodeve’s actual direct labor rate? A . P3 .6 0 C. P 3. 80 B. P4.00 D. P5.80

 The company uses a variable overhead rate of 20% of standard direct-labor cost for flexible budgeting purposes. Actual variable overhead for the month was P30,750. What were the standard hours allowed during the month of April? A. 50,250 C. 58,625 B. 48,550 D. 37,520 xxxiii

. Information on Barber Company’s direct labor costs for the month of January is as follows: Actual direct labor hours 34,500 Standard direct labor hours 35,000  Total direct labor payroll P241,500 Direct labor efficiency variance – favorable P 3,200 What is Barber’s direct labor rate variance? A. P17,250 U C. P21,000 U B. P20,700 U D. P21,000 F

xxxiv

. STA Company uses a standard cost system. The following information pertains to direct labor costs for the month of June: Standard direct labor rate per hour P 10.00 Actual direct labor rate per hour P 9.00 Labor rate variance (favorable) P12,000 Actual output (units) 2,000 Standard hours allowed for actual production 10,000 hours How many actual labor hours were worked during March for STA Company? A. 10,000 C. 8,000 B. 12,000 D. 10,500

Standard Costs and Variance Analysis xxv

P77,775 in direct labor expense. Direct labor hours of 6,375 were actually worked during the month. Variance analysis of the performance for the month of May would show a(n): A. Favorable material quantity variance of P7,500. B. Unfavorable direct labor efficiency variance of P1,275. C. Unfavorable material quantity variance of P7,500. D. Unfavorable direct labor rate variance of P1,275.

. Hingis had a P750 unfavorable direct labor rate variance and an P800 favorable efficiency variance. Hingis paid P7,150 for 800 hours of labor. What was the standard direct labor wage rate? A . P8. 94 C. P 8. 00 B. P7.94 D. P7.80

xxvi

.Powerless Company’s operations for April disclosed the following data relating to direct labor: Actual cost P10,000 Rate variance 1,000 favorable Efficiency variance 1,500 unfavorable Standard cost P 9,500 Actual direct labor hours for April amounted to 2,000. Powerless’ standard direct labor rate per hour in April was: A . P5. 50 C. P 5. 00 B. P4.75 D. P4.50

xxii

. The standard hourly rate was P4.10. Standard hours for the level of  production are 4,000. The actual rate was P4.27. The labor rate variance was P654.50, unfavorable. What were the actual labor hours? A . 3, 700 C. 3 ,8 50 B. 4,150 D. 4,000

xxiii

. Clean Harry Corp. uses two different types of labor to manufacture its product. The types of labor, Mixing and Finishing, have the following standards: Labor Type Standard Mix Std Hourly Standard Rate Cost Mixing 500 hours P10 P5,000 Finishing 250 hours P5 P1,250  Yield: 4,000 units During January, the following actual production information was provided: Labor Type Actual Mix Mixing 4,500 hours Finishing 3,000 hours  Yield: 36,000 units What is the labor mix variance? A. P2,500 F C. P2,500 U B. P5,000 F D. P5,000 F

xxvii

. Lion Company’s direct labor costs for the month of January were as follows: Actual direct labor hours 20,000 Standard direct labor hours 21,000 Direct labor rate variance – Unfav. P 3,000  Total payroll P126,000 What was Lion’s direct labor efficiency variance? A. P6,000 favorable C. P6,150 favorable B. P6,300 favorable D. P6,450 favorable

xxviii

. Using the information given below, determine efficiency variance: Labor price per hour Standard labor price per gallon of output at 20 gal./hr Standard labor cost of 8,440 gallons of actual output Actual total inputs(410 hours at P21/hr) A. P 410 unfavorable C. P 240 favorable B. P 170 unfavorable D. P 410 favorable

xxiv

.How much labor yield variances should be reported? A. P6,250 U C. P5,250 F B. P6,250 F D. P5,250 U

the

labor

P 20 P 1 P8,440 P8,610

211

Standard Costs and Variance Analysis xxix

.Simbad Company’s operations for the month just ended originally set up a 60,000 direct labor hour level, with budgeted direct labor of P960,000 and budgeted variable overhead of P240,000. The actual results revealed that direct labor incurred amounted to P1,148,000 and that the unfavorable variable overhead variance was P40,000. Labor trouble caused an unfavorable labor efficiency variance of P120,000, and new employees hired at higher rates resulted in an actual average wage rate of P16.40 per hour. The total number of standard direct labor hours allowed for the actual units produced is A. P52,500 C. P62,500 B. P77,500 D. P70,000

 The company uses a variable overhead rate of 20% of standard direct-labor cost for flexible budgeting purposes. Actual variable overhead for the month was P30,750. What were the standard hours allowed during the month of April? A. 50,250 C. 58,625 B. 48,550 D. 37,520 xxxiii

. Information on Barber Company’s direct labor costs for the month of January is as follows: Actual direct labor hours 34,500 Standard direct labor hours 35,000  Total direct labor payroll P241,500 Direct labor efficiency variance – favorable P 3,200 What is Barber’s direct labor rate variance? A. P17,250 U C. P21,000 U B. P20,700 U D. P21,000 F

Questions 30 and 31 are based on the following information. Information on Goodeve Company’s direct labor costs are presented below: Standard direct labor hours 30,000 Actual direct labor hours 29,000 Direct labor efficiency variance Favorable P 4,000 Direct labor rate variance Favorable P 5,800  Total payroll P110,200

xxxiv

. STA Company uses a standard cost system. The following information pertains to direct labor costs for the month of June: Standard direct labor rate per hour P 10.00 Actual direct labor rate per hour P 9.00 Labor rate variance (favorable) P12,000 Actual output (units) 2,000 Standard hours allowed for actual production 10,000 hours How many actual labor hours were worked during March for STA Company? A. 10,000 C. 8,000 B. 12,000 D. 10,500

xxx

. What was Goodeve’s standard direct labor rate? A. P3.54 C. P3.80 B . P4 .0 0 D. P5.80

xxxi

.What was Goodeve’s actual direct labor rate? A . P3 .6 0 C. P 3. 80 B. P4.00 D. P5.80 xxxv

. Information of Hanes’ direct labor costs for the month of May is as follows: Actual direct labor rate P7.50 Standard direct labor hours allowed 11,000 Actual direct labor hours 10,000 Direct labor rate variance – favorable P5,500 What was the standard direct labor rate in effect for the month of  May? A. P6.95 C. P8.00

xxxii

. The Islander Corporation makes a variety of leather goods. It uses standards costs and a flexible budget to aid planning and control. Budgeted variable overhead at a 45,000-direct labor hour level is P27,000. During April material purchases were P241,900. Actual direct-labor costs incurred were P140,700. The direct-labor usage variance was P5,100 unfavorable. The actual average wage rate was P0.20 lower than the average standard wage rate. 212

Standard Costs and Variance Analysis

B . P7 .0 0

D. P8.05

Two-way overhead variance xxxvi . The overhead variances for Big Company were: Variable overhead spending variance: P3600 favorable. Variable overhead efficiency variance: P6,000 unfavorable. Fixed overhead spending variance: P10,000 favorable. Fixed overhead volume variance: P24,000 favorable. What was the overhead controllable variance? A. P31,600 favorable C. P24,000 favorable B. P13,600 favorable D. P 7,600 favorable

Using the two-way analysis of overhead variance, what is the controllable variance for December? A. P 3,000 Favorable C. P 5 ,000 Favorable B. P 9,000 Favorable D. P10,500 Unfavorable xxxix

. Heart Company uses a flexible budget system and prepared the following information for the year: Percent of Capacity 80 90 Percent Percent Direct labor hours 24,000 27,000 Variable factory overhead P 54,000 P 60,750 Fixed factory overhead P 81,000 P 81,000  Total factory overhead rate P5.625 P5.25 per DLH Heart operated at 80 percent of capacity during the year, but applied factory overhead based on the 90 percent capacity level. Assuming that actual factory overhead was equal to the budgeted amount of overhead, how much was the overhead volume variance for the year? A. P 9,000 unfavorable C. P 9,000 favorable B. P15,750 unfavorable D. P15,750 favorable

xxxvii

.

Kent Company sets the following standards for 2007: Direct labor cost (2 DLH @ P4.50) P 9.00 Manufacturing overhead (2 DLH @ P7.50) 15.00 Kent Company plans to produce its only product equally each month. The annual budget for overhead costs are: Fixed overhead P150,000 Variable overhead 300,000 Normal activity in direct labor hours 60,000 In March, Kent Company produced 2,450 units with actual direct labor hours used of 5,050. Actual overhead costs for the month amounted to P37,245 (Fixed overhead is as budgeted.)  The amount of overhead volume variance for Kent Company is A. P250 unfavorable C. P500 unfavorable B. P750 Unfavorable D. P375 Unfavorable

xxxviii

. Calma Company uses a standard cost system. The following budget, at normal capacity, and the actual results are summarized

xl

. The Fire Company has a standard absorption and flexible budgeting system and uses a two-way analysis of overhead variances. Selected data for the June production activity are: Budgeted fixed factory overhead costs P 64,000 Actual factory overhead 230,000 Variable factory overhead rater per DLH P 5 Standard DLH 32 000

Standard Costs and Variance Analysis xxix

.Simbad Company’s operations for the month just ended originally set up a 60,000 direct labor hour level, with budgeted direct labor of P960,000 and budgeted variable overhead of P240,000. The actual results revealed that direct labor incurred amounted to P1,148,000 and that the unfavorable variable overhead variance was P40,000. Labor trouble caused an unfavorable labor efficiency variance of P120,000, and new employees hired at higher rates resulted in an actual average wage rate of P16.40 per hour. The total number of standard direct labor hours allowed for the actual units produced is A. P52,500 C. P62,500 B. P77,500 D. P70,000

 The company uses a variable overhead rate of 20% of standard direct-labor cost for flexible budgeting purposes. Actual variable overhead for the month was P30,750. What were the standard hours allowed during the month of April? A. 50,250 C. 58,625 B. 48,550 D. 37,520 xxxiii

. Information on Barber Company’s direct labor costs for the month of January is as follows: Actual direct labor hours 34,500 Standard direct labor hours 35,000  Total direct labor payroll P241,500 Direct labor efficiency variance – favorable P 3,200 What is Barber’s direct labor rate variance? A. P17,250 U C. P21,000 U B. P20,700 U D. P21,000 F

Questions 30 and 31 are based on the following information. Information on Goodeve Company’s direct labor costs are presented below: Standard direct labor hours 30,000 Actual direct labor hours 29,000 Direct labor efficiency variance Favorable P 4,000 Direct labor rate variance Favorable P 5,800  Total payroll P110,200

xxxiv

. STA Company uses a standard cost system. The following information pertains to direct labor costs for the month of June: Standard direct labor rate per hour P 10.00 Actual direct labor rate per hour P 9.00 Labor rate variance (favorable) P12,000 Actual output (units) 2,000 Standard hours allowed for actual production 10,000 hours How many actual labor hours were worked during March for STA Company? A. 10,000 C. 8,000 B. 12,000 D. 10,500

xxx

. What was Goodeve’s standard direct labor rate? A. P3.54 C. P3.80 B . P4 .0 0 D. P5.80

xxxi

.What was Goodeve’s actual direct labor rate? A . P3 .6 0 C. P 3. 80 B. P4.00 D. P5.80 xxxv

. Information of Hanes’ direct labor costs for the month of May is as follows: Actual direct labor rate P7.50 Standard direct labor hours allowed 11,000 Actual direct labor hours 10,000 Direct labor rate variance – favorable P5,500 What was the standard direct labor rate in effect for the month of  May? A. P6.95 C. P8.00

xxxii

. The Islander Corporation makes a variety of leather goods. It uses standards costs and a flexible budget to aid planning and control. Budgeted variable overhead at a 45,000-direct labor hour level is P27,000. During April material purchases were P241,900. Actual direct-labor costs incurred were P140,700. The direct-labor usage variance was P5,100 unfavorable. The actual average wage rate was P0.20 lower than the average standard wage rate. 212

Standard Costs and Variance Analysis

B . P7 .0 0

D. P8.05

Using the two-way analysis of overhead variance, what is the controllable variance for December? A. P 3,000 Favorable C. P 5 ,000 Favorable B. P 9,000 Favorable D. P10,500 Unfavorable

Two-way overhead variance xxxvi . The overhead variances for Big Company were: Variable overhead spending variance: P3600 favorable. Variable overhead efficiency variance: P6,000 unfavorable. Fixed overhead spending variance: P10,000 favorable. Fixed overhead volume variance: P24,000 favorable. What was the overhead controllable variance? A. P31,600 favorable C. P24,000 favorable B. P13,600 favorable D. P 7,600 favorable

xxxix

. Heart Company uses a flexible budget system and prepared the following information for the year: Percent of Capacity 80 90 Percent Percent Direct labor hours 24,000 27,000 Variable factory overhead P 54,000 P 60,750 Fixed factory overhead P 81,000 P 81,000  Total factory overhead rate P5.625 P5.25 per DLH Heart operated at 80 percent of capacity during the year, but applied factory overhead based on the 90 percent capacity level. Assuming that actual factory overhead was equal to the budgeted amount of overhead, how much was the overhead volume variance for the year? A. P 9,000 unfavorable C. P 9,000 favorable B. P15,750 unfavorable D. P15,750 favorable

xxxvii

.

Kent Company sets the following standards for 2007: Direct labor cost (2 DLH @ P4.50) P 9.00 Manufacturing overhead (2 DLH @ P7.50) 15.00 Kent Company plans to produce its only product equally each month. The annual budget for overhead costs are: Fixed overhead P150,000 Variable overhead 300,000 Normal activity in direct labor hours 60,000 In March, Kent Company produced 2,450 units with actual direct labor hours used of 5,050. Actual overhead costs for the month amounted to P37,245 (Fixed overhead is as budgeted.)  The amount of overhead volume variance for Kent Company is A. P250 unfavorable C. P500 unfavorable B. P750 Unfavorable D. P375 Unfavorable

xl

. The Fire Company has a standard absorption and flexible budgeting system and uses a two-way analysis of overhead variances. Selected data for the June production activity are: Budgeted fixed factory overhead costs P 64,000 Actual factory overhead 230,000 Variable factory overhead rater per DLH P 5 Standard DLH 32,000 Actual DLH 32,000  The budget (controllable) variance for June is A. P1,000 favorable C. P1,000 unfavorable B. P6,000 favorable D. P6,000 unfavorable

xxxviii

. Calma Company uses a standard cost system. The following budget, at normal capacity, and the actual results are summarized for the month of December: Direct labor hours 24,000 Variable factory OH P 48,000 Fixed factory OH P108,000  Total factory OH per DLH P 6.50 Actual data for December were as follows: Direct labor hours worked 22,000  Total factory OH P147,000 Stan dard DLHs allowed for cap acit y attaine d 21,000

xli

. Sorsogon Company had actual overhead of P14,000 for the year.  The company applied overhead of P13,400. If the overhead budgeted for the standard hours allowed is P15,600, the overhead controllable variance is

213

Standard Costs and Variance Analysis

A. P 600 Favorable B. P2,200 Unfavorable

C. P1,600 Favorable D. P1,600 Unfavorable

xlii

. Compo Co. uses a predetermined factory O/H application rate based on direct labor cost. For the year ended December 31, Compo’s budgeted factory O/H was P600,000, based on a budgeted volume of 50,000 direct labor hours, at a standard direct labor rate of P6 per hour. Actual factory O/H amounted to P620,000, with actual direct labor cost of P325,000. For the year, over-applied factory O/H was A. P20,000 C. P30,000 B. P25,000 D. P50,000

xliii

. The Terrain Company has a standard absorption and flexible budgeting system and uses a two-way analysis of overhead variances. Selected data for the June production activity are: Budgeted fixed factory overhead costs P 6 4,000 Actual factory overhead 230,000 Variable factory overhead rater per DLH P 5 Standard DLH 32,000 Actual DLH 32,000  The budget (controllable) variance for the month of June is: A. P1,000 favorable C. P1,000 unfavorable B. P6,000 favorable D. P6,000 unfavorable

xliv

. The standard factory overhead rate is P10 per direct labor hour (P8 for variable factory overhead and P2 for fixed factory overhead) based on 100% capacity of 30,000 direct labor hours. The standard cost and the actual cost of factory overhead for the production of 

xlv

. The following data are the actual results for Wow Company for the month of May: Actual output 4,500 units Actual variable overhead P360,000 Actual fixed overhead P108,000 Actual machine time 14,000 MH Standard cost and budget information for Wow Company follows: Standard variable overhead rate P6.00 per MH Stand ard quan tity of machine hou rs 3 hou rs pe r un it Budgeted fixed overhead P777,600 per year Budgeted output 4,800 units per month  The overhead efficiency variance is A. P3,000 Favorable C. P3,000 Unfavorable B. P5,400 Favorable D. P5,400 Unfavorable

xlvi

. The Libiran Company produces its only product, Menthol Chewing Gum. The standard overhead cost for one pack of the product follows: Fixed overhead (1.50 hours at P18.00) P27.00 Variable overhead (1.50 hours at P10.00) 15.00  Total application rate P42.00 Libiran uses expected volume of 20,000 units. During the year, Libiran used 31,500 direct labor hours for the production of 20,000 units. Actual overhead costs were P545,000 fixed and P308,700 variable.  The overhead efficiency variance is A. P22,500 Favorable C. P15,000 Favorable B. P22,500 Unfavorable D. P15,000 Unfavorable

Standard Costs and Variance Analysis

B . P7 .0 0

D. P8.05

Using the two-way analysis of overhead variance, what is the controllable variance for December? A. P 3,000 Favorable C. P 5 ,000 Favorable B. P 9,000 Favorable D. P10,500 Unfavorable

Two-way overhead variance xxxvi . The overhead variances for Big Company were: Variable overhead spending variance: P3600 favorable. Variable overhead efficiency variance: P6,000 unfavorable. Fixed overhead spending variance: P10,000 favorable. Fixed overhead volume variance: P24,000 favorable. What was the overhead controllable variance? A. P31,600 favorable C. P24,000 favorable B. P13,600 favorable D. P 7,600 favorable

xxxix

. Heart Company uses a flexible budget system and prepared the following information for the year: Percent of Capacity 80 90 Percent Percent Direct labor hours 24,000 27,000 Variable factory overhead P 54,000 P 60,750 Fixed factory overhead P 81,000 P 81,000  Total factory overhead rate P5.625 P5.25 per DLH Heart operated at 80 percent of capacity during the year, but applied factory overhead based on the 90 percent capacity level. Assuming that actual factory overhead was equal to the budgeted amount of overhead, how much was the overhead volume variance for the year? A. P 9,000 unfavorable C. P 9,000 favorable B. P15,750 unfavorable D. P15,750 favorable

xxxvii

.

Kent Company sets the following standards for 2007: Direct labor cost (2 DLH @ P4.50) P 9.00 Manufacturing overhead (2 DLH @ P7.50) 15.00 Kent Company plans to produce its only product equally each month. The annual budget for overhead costs are: Fixed overhead P150,000 Variable overhead 300,000 Normal activity in direct labor hours 60,000 In March, Kent Company produced 2,450 units with actual direct labor hours used of 5,050. Actual overhead costs for the month amounted to P37,245 (Fixed overhead is as budgeted.)  The amount of overhead volume variance for Kent Company is A. P250 unfavorable C. P500 unfavorable B. P750 Unfavorable D. P375 Unfavorable

xl

. The Fire Company has a standard absorption and flexible budgeting system and uses a two-way analysis of overhead variances. Selected data for the June production activity are: Budgeted fixed factory overhead costs P 64,000 Actual factory overhead 230,000 Variable factory overhead rater per DLH P 5 Standard DLH 32,000 Actual DLH 32,000  The budget (controllable) variance for June is A. P1,000 favorable C. P1,000 unfavorable B. P6,000 favorable D. P6,000 unfavorable

xxxviii

. Calma Company uses a standard cost system. The following budget, at normal capacity, and the actual results are summarized for the month of December: Direct labor hours 24,000 Variable factory OH P 48,000 Fixed factory OH P108,000  Total factory OH per DLH P 6.50 Actual data for December were as follows: Direct labor hours worked 22,000  Total factory OH P147,000 Stan dard DLHs allowed for cap acit y attaine d 21,000

xli

. Sorsogon Company had actual overhead of P14,000 for the year.  The company applied overhead of P13,400. If the overhead budgeted for the standard hours allowed is P15,600, the overhead controllable variance is

213

Standard Costs and Variance Analysis

A. P 600 Favorable B. P2,200 Unfavorable

xlv

C. P1,600 Favorable D. P1,600 Unfavorable

. The following data are the actual results for Wow Company for the month of May: Actual output 4,500 units Actual variable overhead P360,000 Actual fixed overhead P108,000 Actual machine time 14,000 MH Standard cost and budget information for Wow Company follows: Standard variable overhead rate P6.00 per MH Stand ard quan tity of machine hou rs 3 hou rs pe r un it Budgeted fixed overhead P777,600 per year Budgeted output 4,800 units per month  The overhead efficiency variance is A. P3,000 Favorable C. P3,000 Unfavorable B. P5,400 Favorable D. P5,400 Unfavorable

xlii

. Compo Co. uses a predetermined factory O/H application rate based on direct labor cost. For the year ended December 31, Compo’s budgeted factory O/H was P600,000, based on a budgeted volume of 50,000 direct labor hours, at a standard direct labor rate of P6 per hour. Actual factory O/H amounted to P620,000, with actual direct labor cost of P325,000. For the year, over-applied factory O/H was A. P20,000 C. P30,000 B. P25,000 D. P50,000

xliii

. The Terrain Company has a standard absorption and flexible budgeting system and uses a two-way analysis of overhead variances. Selected data for the June production activity are: Budgeted fixed factory overhead costs P 6 4,000 Actual factory overhead 230,000 Variable factory overhead rater per DLH P 5 Standard DLH 32,000 Actual DLH 32,000  The budget (controllable) variance for the month of June is: A. P1,000 favorable C. P1,000 unfavorable B. P6,000 favorable D. P6,000 unfavorable

xlvi

. The Libiran Company produces its only product, Menthol Chewing Gum. The standard overhead cost for one pack of the product follows: Fixed overhead (1.50 hours at P18.00) P27.00 Variable overhead (1.50 hours at P10.00) 15.00  Total application rate P42.00 Libiran uses expected volume of 20,000 units. During the year, Libiran used 31,500 direct labor hours for the production of 20,000 units. Actual overhead costs were P545,000 fixed and P308,700 variable.  The overhead efficiency variance is A. P22,500 Favorable C. P15,000 Favorable B. P22,500 Unfavorable D. P15,000 Unfavorable

xliv

. The standard factory overhead rate is P10 per direct labor hour (P8 for variable factory overhead and P2 for fixed factory overhead) based on 100% capacity of 30,000 direct labor hours. The standard cost and the actual cost of factory overhead for the production of  5,000 units during May were as follows: Standard:25,000 hours at P10 P250,000 Actual: Variable factory overhead 202,500 Fixed factory overhead 60,000 What is the amount of the factory overhead volume variance? A. 12,500 favorable C. 12,500 unfavorable B. 10,000 unfavorable D. 10,000 favorable

xlvii

.Abbey Company produces a single product. Abbey employs a standard cost system and uses a flexible budget to predict overhead costs at various levels of activity. For the most recent year, Abbey used a standard overhead rate equal to P8.50 per direct labor hour. The rate was computed using normal activity. Budgeted overhead costs are P100,000 for 10,000 direct labor hours and P160,000 for 20,000 direct labor hours. During the past year, Abbey generate the following data: Actual production: 1,400 units

Three-way overhead variance 214

Standard Costs and Variance Analysis

Fixed overhead volume variance: P5,000 U Variable overhead efficiency variance: P3,000 F Actual fixed overhead costs: P42,670 Actual variable overhead costs: P82,000  The number of direct labor hours used as normal activity are: A. 16,000 C. 14,000 B. 15,000 D. 13,500

Fixed overhead expenses, budgeted P 7,000 Actual hours 3,500 Standard hours 3,800 Variable overhead rate per DLH P 2.50 Assuming that Tyro uses a three-way analysis of overhead variances, what is the spending variance? A . P 750 F C. P 95 0 F B. P 750 U D. P1,500 U

xlviii

. Using the information presented below, calculate the total overhead spending variance. Budgeted fixed overhead P10,000 Standard variable overhead (2 DLH at P2 per DLH)P4 per unit Actual fixed overhead P10,300 Actual variable overhead P19,500 Budgeted volume (5,000 units x 2 DLH) 10,000 DLH Actual direct labor hours (DLH) 9,500 Units produced 4,500 A. P 500 U C. P1,000 U B . P 800 U D. P1,300 U

li

.

The Sacto Co.’s standard fixed overhead cost is P3 per direct labor hour based on budgeted fixed costs of P300,000. The standard allows 2 direct labor hours per unit. During 2006, Sacto produced 55,000 units of product, incurred P315,000 of fixed overhead costs, and recorded P106,000 actual hours of direct labor. What are the fixed overhead variances? Spen ding v ariance Volu me variance A. P15,000 U P30,000 F B. P33,000 U P30,000 F C. P15,000 U P18,000 F D. P33,000 U P18,000 F

xlix

. The following data are the actual results for Bustos Company for the month of May: Actual output 4,500 units Actual variable overhead P360,000 Actual fixed overhead P108,000 Actual machine time 14,000 MH Standard cost and budget information for Bustos Company follows: Standard variable overhead rate P6.00 per MH Stan dard qu antity of machine hours 3 hou rs pe r un it

lii

. Using the information in the preceding number, the amounts of  controllable variances for variable overhead are: Spending Efficiency A. P20,000 Fav P20,000 Unf   B. P20,000 Unf P20,000 Fav C. P 5,000 Unf P20,000 Unf   D. P20,000 Fav P 5,000 Unf  

Standard Costs and Variance Analysis

A. P 600 Favorable B. P2,200 Unfavorable

xlv

C. P1,600 Favorable D. P1,600 Unfavorable

. The following data are the actual results for Wow Company for the month of May: Actual output 4,500 units Actual variable overhead P360,000 Actual fixed overhead P108,000 Actual machine time 14,000 MH Standard cost and budget information for Wow Company follows: Standard variable overhead rate P6.00 per MH Stand ard quan tity of machine hou rs 3 hou rs pe r un it Budgeted fixed overhead P777,600 per year Budgeted output 4,800 units per month  The overhead efficiency variance is A. P3,000 Favorable C. P3,000 Unfavorable B. P5,400 Favorable D. P5,400 Unfavorable

xlii

. Compo Co. uses a predetermined factory O/H application rate based on direct labor cost. For the year ended December 31, Compo’s budgeted factory O/H was P600,000, based on a budgeted volume of 50,000 direct labor hours, at a standard direct labor rate of P6 per hour. Actual factory O/H amounted to P620,000, with actual direct labor cost of P325,000. For the year, over-applied factory O/H was A. P20,000 C. P30,000 B. P25,000 D. P50,000

xliii

. The Terrain Company has a standard absorption and flexible budgeting system and uses a two-way analysis of overhead variances. Selected data for the June production activity are: Budgeted fixed factory overhead costs P 6 4,000 Actual factory overhead 230,000 Variable factory overhead rater per DLH P 5 Standard DLH 32,000 Actual DLH 32,000  The budget (controllable) variance for the month of June is: A. P1,000 favorable C. P1,000 unfavorable B. P6,000 favorable D. P6,000 unfavorable

xlvi

. The Libiran Company produces its only product, Menthol Chewing Gum. The standard overhead cost for one pack of the product follows: Fixed overhead (1.50 hours at P18.00) P27.00 Variable overhead (1.50 hours at P10.00) 15.00  Total application rate P42.00 Libiran uses expected volume of 20,000 units. During the year, Libiran used 31,500 direct labor hours for the production of 20,000 units. Actual overhead costs were P545,000 fixed and P308,700 variable.  The overhead efficiency variance is A. P22,500 Favorable C. P15,000 Favorable B. P22,500 Unfavorable D. P15,000 Unfavorable

xliv

. The standard factory overhead rate is P10 per direct labor hour (P8 for variable factory overhead and P2 for fixed factory overhead) based on 100% capacity of 30,000 direct labor hours. The standard cost and the actual cost of factory overhead for the production of  5,000 units during May were as follows: Standard:25,000 hours at P10 P250,000 Actual: Variable factory overhead 202,500 Fixed factory overhead 60,000 What is the amount of the factory overhead volume variance? A. 12,500 favorable C. 12,500 unfavorable B. 10,000 unfavorable D. 10,000 favorable

xlvii

.Abbey Company produces a single product. Abbey employs a standard cost system and uses a flexible budget to predict overhead costs at various levels of activity. For the most recent year, Abbey used a standard overhead rate equal to P8.50 per direct labor hour. The rate was computed using normal activity. Budgeted overhead costs are P100,000 for 10,000 direct labor hours and P160,000 for 20,000 direct labor hours. During the past year, Abbey generate the following data: Actual production: 1,400 units

Three-way overhead variance 214

Standard Costs and Variance Analysis

Fixed overhead volume variance: P5,000 U Variable overhead efficiency variance: P3,000 F Actual fixed overhead costs: P42,670 Actual variable overhead costs: P82,000  The number of direct labor hours used as normal activity are: A. 16,000 C. 14,000 B. 15,000 D. 13,500

Fixed overhead expenses, budgeted P 7,000 Actual hours 3,500 Standard hours 3,800 Variable overhead rate per DLH P 2.50 Assuming that Tyro uses a three-way analysis of overhead variances, what is the spending variance? A . P 750 F C. P 95 0 F B. P 750 U D. P1,500 U

xlviii

. Using the information presented below, calculate the total overhead spending variance. Budgeted fixed overhead P10,000 Standard variable overhead (2 DLH at P2 per DLH)P4 per unit Actual fixed overhead P10,300 Actual variable overhead P19,500 Budgeted volume (5,000 units x 2 DLH) 10,000 DLH Actual direct labor hours (DLH) 9,500 Units produced 4,500 A. P 500 U C. P1,000 U B . P 800 U D. P1,300 U

li

.

The Sacto Co.’s standard fixed overhead cost is P3 per direct labor hour based on budgeted fixed costs of P300,000. The standard allows 2 direct labor hours per unit. During 2006, Sacto produced 55,000 units of product, incurred P315,000 of fixed overhead costs, and recorded P106,000 actual hours of direct labor. What are the fixed overhead variances? Spen ding v ariance Volu me variance A. P15,000 U P30,000 F B. P33,000 U P30,000 F C. P15,000 U P18,000 F D. P33,000 U P18,000 F

xlix

. The following data are the actual results for Bustos Company for the month of May: Actual output 4,500 units Actual variable overhead P360,000 Actual fixed overhead P108,000 Actual machine time 14,000 MH Standard cost and budget information for Bustos Company follows: Standard variable overhead rate P6.00 per MH Stan dard qu antity of machine hours 3 hou rs pe r un it Budgeted fixed overhead P777,600 per year Budgeted output 4,800 unit per month  The overhead efficiency variance is: A. P3,000 Favorable C. P3,000 Unfavorable B. P5,400 Favorable D. P5,400 Unfavorable

l

.

lii

. Using the information in the preceding number, the amounts of  controllable variances for variable overhead are: Spending Efficiency A. P20,000 Fav P20,000 Unf   B. P20,000 Unf P20,000 Fav C. P 5,000 Unf P20,000 Unf   D. P20,000 Fav P 5,000 Unf  

Four-way overhead variance liii . Safin Corporation’s master budget calls for the production of 5,000 units of product monthly. The annual master budget includes indirect labor of P144,000 annually. Safin considers indirect labor to be a variable cost. During the month of April, 4,500 units of  product were produced, and indirect labor costs of P10,100 were incurred. A performance report utilizing flexible budgeting would report a budget variance for indirect labor of: A. P1,900 Unfavorable. C. P1,900 Favorable.

The following information is available from the Tyro Company: Actual factory overhead P15,000 Fixed overhead expenses, actual P 7,200 215

Standard Costs and Variance Analysis

B. P 700 Unfavorable.

D. P 700 Favorable.

A . P1. 28 B. P1.25

C. P 1. 39 D. P1.52

liv

. Wala Company applies overhead on a direct labor hour basis. Each unit of product requires 5 direct labor hours. Overhead is applied on a 30 percent variable and 70 percent fixed basis; the overhead application rate is P16 per hour. Standards are based on a normal monthly capacity of 5,000 direct labor hours. During September 2006, Wala produced 1,010 units of product and incurred 4,900 direct labor hours. Actual overhead cost for the month was P80,000. What is total annual budgeted fixed overhead cost? A. P 56,000 C. P672,000 B. P 56,560 D. P678,720

lviii

. Calvin Klein Company has a standard fixed cost of P6 per unit. At an actual production of 8,000 units a favorable volume variance of  P12,000 resulted. What were total budgeted fixed costs? A. P36,000. C. P48,000. B. P60,000. D. P75,000.

lix

. Puma Company had an 25,000 unfavorable volume variance, a P18,000 unfavorable variable overhead spending variance, and P2,000 total under applied overhead. The fixed overhead budget variance is A. P41,000 favorable C. P45,000 favorable B. P41,000 Unfavorable D. P45,000 Unfavorable

lv

. Budgeted variable overhead for the level of production achieved is 40,000 machine-hours at a budgeted cost of P62,000. Actual variable overhead at the level of production achieved was 38,000 hours at an actual cost of P62,400. What is the total variable overhead variance? A. P400 favorable C. P3,100 unfavorable B. P400 unfavorable D. P3,100 favorable

lvi

. The Pinatubo Company makes and sells a single product and uses standard costing. During January, the company actually used 8,700 direct labor-hours (DLHs) and produced 3,000 units of product. The standard cost card for one unit of product includes the following: Variable factory overhead: 3.0 DLHs @ P4.00 per DLH Fixed factory overhead: 3.0 DLHs @ P3.50 per DLH For January, the company incurred P22,000 of actual fixed overhead

lx

. Arlene had an P18,000 unfavorable volume variance, a P25,000 unfavorable variable overhead spending variance, and P2,000 total under applied overhead. The fixed overhead budget variance is: A. P41,000 favorable C. P45,000 favorable B. P41,000 Unfavorable D. P45,000 Unfavorable

lxi

. Fixed manufacturing overhead was budgeted at P500,000 and 25,000 direct labor hours were budgeted. If the fixed overhead volume variance was P12,000 favorable and the fixed overhead spending variance was P16,000 unfavorable, fixed manufacturing overhead applied must be: A. P516,000 C. P512,000 B. P504,000 D. P496,000

Standard Costs and Variance Analysis

Fixed overhead volume variance: P5,000 U Variable overhead efficiency variance: P3,000 F Actual fixed overhead costs: P42,670 Actual variable overhead costs: P82,000  The number of direct labor hours used as normal activity are: A. 16,000 C. 14,000 B. 15,000 D. 13,500

Fixed overhead expenses, budgeted P 7,000 Actual hours 3,500 Standard hours 3,800 Variable overhead rate per DLH P 2.50 Assuming that Tyro uses a three-way analysis of overhead variances, what is the spending variance? A . P 750 F C. P 95 0 F B. P 750 U D. P1,500 U

xlviii

. Using the information presented below, calculate the total overhead spending variance. Budgeted fixed overhead P10,000 Standard variable overhead (2 DLH at P2 per DLH)P4 per unit Actual fixed overhead P10,300 Actual variable overhead P19,500 Budgeted volume (5,000 units x 2 DLH) 10,000 DLH Actual direct labor hours (DLH) 9,500 Units produced 4,500 A. P 500 U C. P1,000 U B . P 800 U D. P1,300 U

li

.

The Sacto Co.’s standard fixed overhead cost is P3 per direct labor hour based on budgeted fixed costs of P300,000. The standard allows 2 direct labor hours per unit. During 2006, Sacto produced 55,000 units of product, incurred P315,000 of fixed overhead costs, and recorded P106,000 actual hours of direct labor. What are the fixed overhead variances? Spen ding v ariance Volu me variance A. P15,000 U P30,000 F B. P33,000 U P30,000 F C. P15,000 U P18,000 F D. P33,000 U P18,000 F

xlix

. The following data are the actual results for Bustos Company for the month of May: Actual output 4,500 units Actual variable overhead P360,000 Actual fixed overhead P108,000 Actual machine time 14,000 MH Standard cost and budget information for Bustos Company follows: Standard variable overhead rate P6.00 per MH Stan dard qu antity of machine hours 3 hou rs pe r un it Budgeted fixed overhead P777,600 per year Budgeted output 4,800 unit per month  The overhead efficiency variance is: A. P3,000 Favorable C. P3,000 Unfavorable B. P5,400 Favorable D. P5,400 Unfavorable

l

.

lii

. Using the information in the preceding number, the amounts of  controllable variances for variable overhead are: Spending Efficiency A. P20,000 Fav P20,000 Unf   B. P20,000 Unf P20,000 Fav C. P 5,000 Unf P20,000 Unf   D. P20,000 Fav P 5,000 Unf  

Four-way overhead variance liii . Safin Corporation’s master budget calls for the production of 5,000 units of product monthly. The annual master budget includes indirect labor of P144,000 annually. Safin considers indirect labor to be a variable cost. During the month of April, 4,500 units of  product were produced, and indirect labor costs of P10,100 were incurred. A performance report utilizing flexible budgeting would report a budget variance for indirect labor of: A. P1,900 Unfavorable. C. P1,900 Favorable.

The following information is available from the Tyro Company: Actual factory overhead P15,000 Fixed overhead expenses, actual P 7,200 215

Standard Costs and Variance Analysis

B. P 700 Unfavorable.

D. P 700 Favorable.

A . P1. 28 B. P1.25

C. P 1. 39 D. P1.52

liv

. Wala Company applies overhead on a direct labor hour basis. Each unit of product requires 5 direct labor hours. Overhead is applied on a 30 percent variable and 70 percent fixed basis; the overhead application rate is P16 per hour. Standards are based on a normal monthly capacity of 5,000 direct labor hours. During September 2006, Wala produced 1,010 units of product and incurred 4,900 direct labor hours. Actual overhead cost for the month was P80,000. What is total annual budgeted fixed overhead cost? A. P 56,000 C. P672,000 B. P 56,560 D. P678,720

lviii

. Calvin Klein Company has a standard fixed cost of P6 per unit. At an actual production of 8,000 units a favorable volume variance of  P12,000 resulted. What were total budgeted fixed costs? A. P36,000. C. P48,000. B. P60,000. D. P75,000.

lix

. Puma Company had an 25,000 unfavorable volume variance, a P18,000 unfavorable variable overhead spending variance, and P2,000 total under applied overhead. The fixed overhead budget variance is A. P41,000 favorable C. P45,000 favorable B. P41,000 Unfavorable D. P45,000 Unfavorable

lv

. Budgeted variable overhead for the level of production achieved is 40,000 machine-hours at a budgeted cost of P62,000. Actual variable overhead at the level of production achieved was 38,000 hours at an actual cost of P62,400. What is the total variable overhead variance? A. P400 favorable C. P3,100 unfavorable B. P400 unfavorable D. P3,100 favorable

lx

. Arlene had an P18,000 unfavorable volume variance, a P25,000 unfavorable variable overhead spending variance, and P2,000 total under applied overhead. The fixed overhead budget variance is: A. P41,000 favorable C. P45,000 favorable B. P41,000 Unfavorable D. P45,000 Unfavorable

lvi

lxi

. The Pinatubo Company makes and sells a single product and uses standard costing. During January, the company actually used 8,700 direct labor-hours (DLHs) and produced 3,000 units of product. The standard cost card for one unit of product includes the following: Variable factory overhead: 3.0 DLHs @ P4.00 per DLH Fixed factory overhead: 3.0 DLHs @ P3.50 per DLH For January, the company incurred P22,000 of actual fixed overhead costs and recorded a P875 favorable volume variance.  The budgeted fixed overhead cost for January is: A. P31,500 C. P30,625 B. P32,375 D. P33,250

. Fixed manufacturing overhead was budgeted at P500,000 and 25,000 direct labor hours were budgeted. If the fixed overhead volume variance was P12,000 favorable and the fixed overhead spending variance was P16,000 unfavorable, fixed manufacturing overhead applied must be: A. P516,000 C. P512,000 B. P504,000 D. P496,000

lxii

. CTV Company has a standard fixed cost of P6 per unit. At an actual production of 8,000 units a favorable volume variance of P12,000 resulted. What were total budgeted fixed costs? A. P36,000 C. P48,000 B. P60,000 D. P75,000

lvii

. The variable-overhead spending variance is P1,080, unfavorable. Variable overhead budgeted at 40,000 machine hours is P50,000. Actual machine hours were 36,000. What was the actual variableoverhead rate per machine hour?

lxiii

. Richard Company employs a standard absorption system for product costing. The standard cost of its product is as follows:

216

Standard Costs and Variance Analysis

Raw materials P14.50 Direct labor (2 DLH x P8) 16.00 Manufacturing overhead (2 DLH x P11) 22.00  Total standard cost P52.50  The manufacturing overhead rate is based upon a normal activity level of 600,000 direct labor hours. Richard planned to produce 25,000 units each month during the year. The budgeted annual manufacturing overhead is: Variable P 3,600,000 Fixed 3,000,000 P 6,600,000 During November, Richard produced 26,000 units. Richard used 53,500 direct labor hours in November at a cost of P433,350. Actual manufacturing overhead for the month was P260,000 fixed and P315,000 variable. The total manufacturing overhead applied during November was P572,000.  The fixed manufacturing overhead volume variance for November is: A. P10,000 favorable C. P10,000 unfavorable B. P3,000 unfavorable D. P22,000 favorable lxiv

. Using the information for Richard Company in the preceding number, the total variance related to efficiency of the manufacturing operation for November is: A. P 9,000 unfavorable C. P12,000 unfavorable B. P21,000 unfavorable D. P11,000 unfavorable

Question Nos. 65 and 66 are based on the following:  Tiny Bubbles Company had the following activity relating to its fixed

Applied (Standard input allowed for actual output achieved x the budgeted rate) Fixed overhead P125,000 Variable overh ead spen din g variance 1,200 F Production volume variance 5,000 U lxv

. If the budgeted rate for applying variable manufacturing overhead was P20 per direct labor hour, how efficient or inefficient was Tiny Bubbles in terms of using direct labor hours as an activity base? A. 100 direct labor hours inefficient C. 100 direct labor hours efficient B. 440 direct labor hours inefficient D. 440 dire ct labor hours efficient

lxvi

. The fixed overhead efficiency variance is: A. P 3,000 favorable C. P 3,000 unfavorable B. P10,000 unfavorable D. Never a meaningful variance

Questions 67 and 68 are based on a monthly normal volume of  50,000 units (100,000 direct labor hours). Raff Co.’s standard cost system contains the following overhead costs: Variable P6 per unit Fixed P8 per unit  The following information pertains to the month of March: Units actually produced Actual direct labor hours worked

38,000 80,000

Standard Costs and Variance Analysis

B. P 700 Unfavorable.

D. P 700 Favorable.

A . P1. 28 B. P1.25

C. P 1. 39 D. P1.52

liv

. Wala Company applies overhead on a direct labor hour basis. Each unit of product requires 5 direct labor hours. Overhead is applied on a 30 percent variable and 70 percent fixed basis; the overhead application rate is P16 per hour. Standards are based on a normal monthly capacity of 5,000 direct labor hours. During September 2006, Wala produced 1,010 units of product and incurred 4,900 direct labor hours. Actual overhead cost for the month was P80,000. What is total annual budgeted fixed overhead cost? A. P 56,000 C. P672,000 B. P 56,560 D. P678,720

lviii

. Calvin Klein Company has a standard fixed cost of P6 per unit. At an actual production of 8,000 units a favorable volume variance of  P12,000 resulted. What were total budgeted fixed costs? A. P36,000. C. P48,000. B. P60,000. D. P75,000.

lix

. Puma Company had an 25,000 unfavorable volume variance, a P18,000 unfavorable variable overhead spending variance, and P2,000 total under applied overhead. The fixed overhead budget variance is A. P41,000 favorable C. P45,000 favorable B. P41,000 Unfavorable D. P45,000 Unfavorable

lv

. Budgeted variable overhead for the level of production achieved is 40,000 machine-hours at a budgeted cost of P62,000. Actual variable overhead at the level of production achieved was 38,000 hours at an actual cost of P62,400. What is the total variable overhead variance? A. P400 favorable C. P3,100 unfavorable B. P400 unfavorable D. P3,100 favorable

lx

. Arlene had an P18,000 unfavorable volume variance, a P25,000 unfavorable variable overhead spending variance, and P2,000 total under applied overhead. The fixed overhead budget variance is: A. P41,000 favorable C. P45,000 favorable B. P41,000 Unfavorable D. P45,000 Unfavorable

lvi

lxi

. The Pinatubo Company makes and sells a single product and uses standard costing. During January, the company actually used 8,700 direct labor-hours (DLHs) and produced 3,000 units of product. The standard cost card for one unit of product includes the following: Variable factory overhead: 3.0 DLHs @ P4.00 per DLH Fixed factory overhead: 3.0 DLHs @ P3.50 per DLH For January, the company incurred P22,000 of actual fixed overhead costs and recorded a P875 favorable volume variance.  The budgeted fixed overhead cost for January is: A. P31,500 C. P30,625 B. P32,375 D. P33,250

. Fixed manufacturing overhead was budgeted at P500,000 and 25,000 direct labor hours were budgeted. If the fixed overhead volume variance was P12,000 favorable and the fixed overhead spending variance was P16,000 unfavorable, fixed manufacturing overhead applied must be: A. P516,000 C. P512,000 B. P504,000 D. P496,000

lxii

. CTV Company has a standard fixed cost of P6 per unit. At an actual production of 8,000 units a favorable volume variance of P12,000 resulted. What were total budgeted fixed costs? A. P36,000 C. P48,000 B. P60,000 D. P75,000

lvii

. The variable-overhead spending variance is P1,080, unfavorable. Variable overhead budgeted at 40,000 machine hours is P50,000. Actual machine hours were 36,000. What was the actual variableoverhead rate per machine hour?

lxiii

. Richard Company employs a standard absorption system for product costing. The standard cost of its product is as follows:

216

Standard Costs and Variance Analysis

Raw materials P14.50 Direct labor (2 DLH x P8) 16.00 Manufacturing overhead (2 DLH x P11) 22.00  Total standard cost P52.50  The manufacturing overhead rate is based upon a normal activity level of 600,000 direct labor hours. Richard planned to produce 25,000 units each month during the year. The budgeted annual manufacturing overhead is: Variable P 3,600,000 Fixed 3,000,000 P 6,600,000 During November, Richard produced 26,000 units. Richard used 53,500 direct labor hours in November at a cost of P433,350. Actual manufacturing overhead for the month was P260,000 fixed and P315,000 variable. The total manufacturing overhead applied during November was P572,000.  The fixed manufacturing overhead volume variance for November is: A. P10,000 favorable C. P10,000 unfavorable B. P3,000 unfavorable D. P22,000 favorable

Applied (Standard input allowed for actual output achieved x the budgeted rate) Fixed overhead P125,000 Variable overh ead spen din g variance 1,200 F Production volume variance 5,000 U lxv

. If the budgeted rate for applying variable manufacturing overhead was P20 per direct labor hour, how efficient or inefficient was Tiny Bubbles in terms of using direct labor hours as an activity base? A. 100 direct labor hours inefficient C. 100 direct labor hours efficient B. 440 direct labor hours inefficient D. 440 dire ct labor hours efficient

lxvi

. The fixed overhead efficiency variance is: A. P 3,000 favorable C. P 3,000 unfavorable B. P10,000 unfavorable D. Never a meaningful variance

Questions 67 and 68 are based on a monthly normal volume of  50,000 units (100,000 direct labor hours). Raff Co.’s standard cost system contains the following overhead costs: Variable P6 per unit Fixed P8 per unit

lxiv

. Using the information for Richard Company in the preceding number, the total variance related to efficiency of the manufacturing operation for November is: A. P 9,000 unfavorable C. P12,000 unfavorable B. P21,000 unfavorable D. P11,000 unfavorable

 The following information pertains to the month of March: Units actually produced 38,000 Actual direct labor hours worked 80,000 Actual overhead incurred: Variable P250,000 Fixed 384,000

Question Nos. 65 and 66 are based on the following:  Tiny Bubbles Company had the following activity relating to its fixed and variable overhead for the month of July. Actual costs Fixed overhead P120,000 Variable overhead 80,000

lxvii

. For March, the unfavorable variable overhead spending variance was: A. P6,000 C. P12,000 B. P10,000 D. P22,000

Flexible budget (Standard input allowed for actual output achieved x the budgeted rate) Variable overhead P 90,000 217

Standard Costs and Variance Analysis lxviii

. For March, the fixed overhead volume variance was: A. P96,000U C. P80,000U B. P96,000F D. P80,000F

lxix

. Edney Company employs standard absorption system for product costing. The standard cost of its product is as follows: Raw materials P14.50 Direct labor (2 DLH x P8) 16.00 Manufacturing overhead (2 DLH x P11) 22.00  The manufacturing overhead rate is based upon a normal activity level of 600,000 direct labor hours. Edney planned to produce 25,000 units each month during the year. The budgeted annual manufacturing overhead is Variable P3,600,000 Fixed 3,000,000 During November, Edney produced 26,000 units. Edney used 53,500 direct labor hours in November at a cost of P433,350. Actual manufacturing overhead for the month was P260,000 fixed and P315,000 variable. The total manufacturing overhead applied during November was P572,000.  The variable manufacturing overhead variances for November are: Spending Efficiency A . P9 ,0 00 u nf av or ab le P 3, 000 un fa vo ra ble B. P6,000 favorable P9,000 unfavorable C. P4,000 unf avorable P1,000 favorable D. P9,000 favorable P12,000 unfavorable

lxx

. The fixed manufacturing overhead variances for November are: Spending Volume

month of May, 2006: Budgeted Actual Units produced 40,000 38,000 Variable manufacturing OH P 4/DLH P16,400 Fixed manufacturing overhead P20/DLH P88,000 Direct labor hours 6 min/unit 4,200 hr lxxi . What is the fixed overhead spending variance? A. P4,000 Favorable C. P8,000 Unfavorable B. P8,000 Favorable D. P4,000 Unfavorable lxxii

. What is the volume variance? A. P4,000 Favorable B. P4,000 Unfavorable

C. P8,000 Favorable D. P8,000 Unfavorable

lxxiii

. How much was the variable overhead spending variance? A. P 400 Favorable C. P400 Unfavorable B. P1,200 Favorable. D. P1,200 Unfavorable

lxxiv

. How much overhead efficiency variance resulted for the month of May? A. P1,600 Favorable C. P1,600 Unfavorable B. P 800 Favorable D. P800 Unfavorable

Questions 75 through 78 are based on Darf Company, which applies overhead on the basis of direct labor hours. Two direct labor hours are required for each product unit. Planned production for the period was set at 9,000 units. Manufacturing overhead is budgeted at P135,000 for the period, of which 20% of this cost is fixed. The 17,200 hours worked during the period resulted in production of 8,500 units.

Standard Costs and Variance Analysis

Raw materials P14.50 Direct labor (2 DLH x P8) 16.00 Manufacturing overhead (2 DLH x P11) 22.00  Total standard cost P52.50  The manufacturing overhead rate is based upon a normal activity level of 600,000 direct labor hours. Richard planned to produce 25,000 units each month during the year. The budgeted annual manufacturing overhead is: Variable P 3,600,000 Fixed 3,000,000 P 6,600,000 During November, Richard produced 26,000 units. Richard used 53,500 direct labor hours in November at a cost of P433,350. Actual manufacturing overhead for the month was P260,000 fixed and P315,000 variable. The total manufacturing overhead applied during November was P572,000.  The fixed manufacturing overhead volume variance for November is: A. P10,000 favorable C. P10,000 unfavorable B. P3,000 unfavorable D. P22,000 favorable

Applied (Standard input allowed for actual output achieved x the budgeted rate) Fixed overhead P125,000 Variable overh ead spen din g variance 1,200 F Production volume variance 5,000 U lxv

. If the budgeted rate for applying variable manufacturing overhead was P20 per direct labor hour, how efficient or inefficient was Tiny Bubbles in terms of using direct labor hours as an activity base? A. 100 direct labor hours inefficient C. 100 direct labor hours efficient B. 440 direct labor hours inefficient D. 440 dire ct labor hours efficient

lxvi

. The fixed overhead efficiency variance is: A. P 3,000 favorable C. P 3,000 unfavorable B. P10,000 unfavorable D. Never a meaningful variance

Questions 67 and 68 are based on a monthly normal volume of  50,000 units (100,000 direct labor hours). Raff Co.’s standard cost system contains the following overhead costs: Variable P6 per unit Fixed P8 per unit

lxiv

. Using the information for Richard Company in the preceding number, the total variance related to efficiency of the manufacturing operation for November is: A. P 9,000 unfavorable C. P12,000 unfavorable B. P21,000 unfavorable D. P11,000 unfavorable

 The following information pertains to the month of March: Units actually produced 38,000 Actual direct labor hours worked 80,000 Actual overhead incurred: Variable P250,000 Fixed 384,000

Question Nos. 65 and 66 are based on the following:  Tiny Bubbles Company had the following activity relating to its fixed and variable overhead for the month of July. Actual costs Fixed overhead P120,000 Variable overhead 80,000

lxvii

. For March, the unfavorable variable overhead spending variance was: A. P6,000 C. P12,000 B. P10,000 D. P22,000

Flexible budget (Standard input allowed for actual output achieved x the budgeted rate) Variable overhead P 90,000 217

Standard Costs and Variance Analysis lxviii

. For March, the fixed overhead volume variance was: A. P96,000U C. P80,000U B. P96,000F D. P80,000F

month of May, 2006: Budgeted Actual Units produced 40,000 38,000 Variable manufacturing OH P 4/DLH P16,400 Fixed manufacturing overhead P20/DLH P88,000 Direct labor hours 6 min/unit 4,200 hr lxxi . What is the fixed overhead spending variance? A. P4,000 Favorable C. P8,000 Unfavorable B. P8,000 Favorable D. P4,000 Unfavorable

lxix

. Edney Company employs standard absorption system for product costing. The standard cost of its product is as follows: Raw materials P14.50 Direct labor (2 DLH x P8) 16.00 Manufacturing overhead (2 DLH x P11) 22.00  The manufacturing overhead rate is based upon a normal activity level of 600,000 direct labor hours. Edney planned to produce 25,000 units each month during the year. The budgeted annual manufacturing overhead is Variable P3,600,000 Fixed 3,000,000 During November, Edney produced 26,000 units. Edney used 53,500 direct labor hours in November at a cost of P433,350. Actual manufacturing overhead for the month was P260,000 fixed and P315,000 variable. The total manufacturing overhead applied during November was P572,000.  The variable manufacturing overhead variances for November are: Spending Efficiency A . P9 ,0 00 u nf av or ab le P 3, 000 un fa vo ra ble B. P6,000 favorable P9,000 unfavorable C. P4,000 unf avorable P1,000 favorable D. P9,000 favorable P12,000 unfavorable

lxxii

. What is the volume variance? A. P4,000 Favorable B. P4,000 Unfavorable

C. P8,000 Favorable D. P8,000 Unfavorable

lxxiii

. How much was the variable overhead spending variance? A. P 400 Favorable C. P400 Unfavorable B. P1,200 Favorable. D. P1,200 Unfavorable

lxxiv

. How much overhead efficiency variance resulted for the month of May? A. P1,600 Favorable C. P1,600 Unfavorable B. P 800 Favorable D. P800 Unfavorable

Questions 75 through 78 are based on Darf Company, which applies overhead on the basis of direct labor hours. Two direct labor hours are required for each product unit. Planned production for the period was set at 9,000 units. Manufacturing overhead is budgeted at P135,000 for the period, of which 20% of this cost is fixed. The 17,200 hours worked during the period resulted in production of 8,500 units. Variable manufacturing overhead cost incurred was P108,500 and fixed manufacturing overhead cost was P28,000. Darf Company uses a four variance method for analyzing manufacturing overhead.

lxx

. The fixed manufacturing overhead variances for November are: Spending Volume A. P10,000 favorable P10,000 favorable B. P10,000 unfavorable P10,000 favorable C. P 6 ,000 favorable P 3 ,000 unfavorable D . P 4, 00 0 un fa vo ra ble P 22, 000 fa vo ra bl e

lxxv

.The variable overhead spending variance for the period is A. P5,300 unfavorable C. P6,300 unfavorable B. P1,200 unfavorable D. P6,500 unfavorable

The following information will be used to answer Question Nos. 71 through 74: Garch, Inc. analyzes manufacturing overhead in the production of its only one product, CD. The following set of information applies to the

lxxvi

.

The variable overhead efficiency variance (quantity) variance for

218

Standard Costs and Variance Analysis

the period is A. P5,300 unfavorable B. P1,500 unfavorable

C. P1,200 unfavorable D. P6,500 unfavorable

lxxvii

. The fixed overhead budget (spending) variance for the period is A. P6,300 unfavorable C. P2,500 unfavorable B. P1,500 unfavorable D. P1,000 unfavorable

lxxviii

. The fixed overhead volume (denominator) variance for the period is A. P 750 unfavorable C. P2,500 unfavorable B. P1,500 unfavorable D. P1,000 unfavorable

Comprehensive lxxix . Big Marat, Inc. began operations on January 3. Standard costs were established in early January assuming a normal production volume of 160,000 units. However, Big Marat produced only 140,000 units of product and sold 100,000 units at a selling price of  P180 per unit during the year. Variable costs totaled P7,000,000, of  which 60% were manufacturing and 40% were selling. Fixed costs totaled P11,200,000, of which 50% were manufacturing and 50% were selling. Big Marat had no raw materials or work-in-process inventories at December 31. Actual input prices and quantities per unit of product were equal to standard. Using absorption costing, Big Marat’s income statement would show: A. B. C. D. C ost o f G oo ds S ol d a t P8,200,0 P7,200,0 P6,500,0 P7,000,00 Standard Cost 00 00 00 0

fact, that yesterday she called you into her office and asked you to attend the executive committee meeting this morning for the purpose of leading a discussion on the variances reported for last period. Anxious to favorably impress the executive committee, you took the variances and supporting data home last night to study. On your way to work this morning, the papers were laying on the seat of your new, red convertible. As you were crossing a bridge on the highway, a sudden gust of wind caught the papers and blew them over the edge of the bridge and into the stream below. You managed to retrieve only one page, which contains the following information: Standard Cost Summary Direct materials, 6 pounds at P3 Direct labor, 0.8 hours at P5 Variable overhead, 0.8 hours at P3 Fixed overhead, 0.8 hours at P7

 Total Standard Cost

Direct materials Direct labor

P405,000 90,000

P18.0 0 4.00 2.40 5.60 P30.0 0

V A R IA N C ES R E P O RT E D Price Spendin Quantit Volume or g or y or Rate Budget Efficienc y P6,90 P9,000 0F U 4,850 7,000 U

Standard Costs and Variance Analysis lxviii

. For March, the fixed overhead volume variance was: A. P96,000U C. P80,000U B. P96,000F D. P80,000F

month of May, 2006: Budgeted Actual Units produced 40,000 38,000 Variable manufacturing OH P 4/DLH P16,400 Fixed manufacturing overhead P20/DLH P88,000 Direct labor hours 6 min/unit 4,200 hr lxxi . What is the fixed overhead spending variance? A. P4,000 Favorable C. P8,000 Unfavorable B. P8,000 Favorable D. P4,000 Unfavorable

lxix

. Edney Company employs standard absorption system for product costing. The standard cost of its product is as follows: Raw materials P14.50 Direct labor (2 DLH x P8) 16.00 Manufacturing overhead (2 DLH x P11) 22.00  The manufacturing overhead rate is based upon a normal activity level of 600,000 direct labor hours. Edney planned to produce 25,000 units each month during the year. The budgeted annual manufacturing overhead is Variable P3,600,000 Fixed 3,000,000 During November, Edney produced 26,000 units. Edney used 53,500 direct labor hours in November at a cost of P433,350. Actual manufacturing overhead for the month was P260,000 fixed and P315,000 variable. The total manufacturing overhead applied during November was P572,000.  The variable manufacturing overhead variances for November are: Spending Efficiency A . P9 ,0 00 u nf av or ab le P 3, 000 un fa vo ra ble B. P6,000 favorable P9,000 unfavorable C. P4,000 unf avorable P1,000 favorable D. P9,000 favorable P12,000 unfavorable

lxxii

. What is the volume variance? A. P4,000 Favorable B. P4,000 Unfavorable

C. P8,000 Favorable D. P8,000 Unfavorable

lxxiii

. How much was the variable overhead spending variance? A. P 400 Favorable C. P400 Unfavorable B. P1,200 Favorable. D. P1,200 Unfavorable

lxxiv

. How much overhead efficiency variance resulted for the month of May? A. P1,600 Favorable C. P1,600 Unfavorable B. P 800 Favorable D. P800 Unfavorable

Questions 75 through 78 are based on Darf Company, which applies overhead on the basis of direct labor hours. Two direct labor hours are required for each product unit. Planned production for the period was set at 9,000 units. Manufacturing overhead is budgeted at P135,000 for the period, of which 20% of this cost is fixed. The 17,200 hours worked during the period resulted in production of 8,500 units. Variable manufacturing overhead cost incurred was P108,500 and fixed manufacturing overhead cost was P28,000. Darf Company uses a four variance method for analyzing manufacturing overhead.

lxx

. The fixed manufacturing overhead variances for November are: Spending Volume A. P10,000 favorable P10,000 favorable B. P10,000 unfavorable P10,000 favorable C. P 6 ,000 favorable P 3 ,000 unfavorable D . P 4, 00 0 un fa vo ra ble P 22, 000 fa vo ra bl e

lxxv

.The variable overhead spending variance for the period is A. P5,300 unfavorable C. P6,300 unfavorable B. P1,200 unfavorable D. P6,500 unfavorable

The following information will be used to answer Question Nos. 71 through 74: Garch, Inc. analyzes manufacturing overhead in the production of its only one product, CD. The following set of information applies to the

lxxvi

.

The variable overhead efficiency variance (quantity) variance for

218

Standard Costs and Variance Analysis

the period is A. P5,300 unfavorable B. P1,500 unfavorable

fact, that yesterday she called you into her office and asked you to attend the executive committee meeting this morning for the purpose of leading a discussion on the variances reported for last period. Anxious to favorably impress the executive committee, you took the variances and supporting data home last night to study.

C. P1,200 unfavorable D. P6,500 unfavorable

lxxvii

. The fixed overhead budget (spending) variance for the period is A. P6,300 unfavorable C. P2,500 unfavorable B. P1,500 unfavorable D. P1,000 unfavorable

On your way to work this morning, the papers were laying on the seat of your new, red convertible. As you were crossing a bridge on the highway, a sudden gust of wind caught the papers and blew them over the edge of the bridge and into the stream below. You managed to retrieve only one page, which contains the following information:

lxxviii

. The fixed overhead volume (denominator) variance for the period is A. P 750 unfavorable C. P2,500 unfavorable B. P1,500 unfavorable D. P1,000 unfavorable

Standard Cost Summary Direct materials, 6 pounds at P3

Comprehensive lxxix . Big Marat, Inc. began operations on January 3. Standard costs were established in early January assuming a normal production volume of 160,000 units. However, Big Marat produced only 140,000 units of product and sold 100,000 units at a selling price of  P180 per unit during the year. Variable costs totaled P7,000,000, of  which 60% were manufacturing and 40% were selling. Fixed costs totaled P11,200,000, of which 50% were manufacturing and 50% were selling. Big Marat had no raw materials or work-in-process inventories at December 31. Actual input prices and quantities per unit of product were equal to standard. Using absorption costing, Big Marat’s income statement would show: A. B. C. D. C ost o f G oo ds S ol d a t P8,200,0 P7,200,0 P6,500,0 P7,000,00 Standard Cost 00 00 00 0 Overhead Volume Variance P800,000P800,000 P700,000 P700,000 U F U F

P18.0 0 4.00 2.40 5.60 P30.0 0

Direct labor, 0.8 hours at P5 Variable overhead, 0.8 hours at P3 Fixed overhead, 0.8 hours at P7

 Total Standard Cost

Direct materials Direct labor

P405,000 90,000

V A R IA N C ES R E P O RT E D Price Spendin Quantit Volume or g or y or Rate Budget Efficienc y P6,90 P9,000 0F U 4,850 7,000 U U P1,300 F ?@

Variable 54,000 overhead Fixed 126,000 500 F overhead Applied to Work in process during the period @ Figure obliterated.

Questions No. 80 through 85 are based on the following information:  You have recently graduated from a university and have accepted a position with Villar Company, the manufacturer of a popular consumer product. During your first week on the job, the vice president has been favorably impressed with your work. She has been so impressed, in

 You

recall

that

manufacturing

overhead

P14,000 U

cost

is

applied

to

219

Standard Costs and Variance Analysis

production on the basis of direct labor-hours and that all of the materials purchased during the period were used in production. Since the company uses JIT to control work flows, work in process inventories are insignificant and can be ignored. It is now 8:30 A.M. The executive committee meeting starts in just one hour; you realize that to avoid looking like a bungling fool you must somehow generate the necessary “backup” data for the variances before the meeting begins. Without backup data it will be impossible to lead the discussion or answer any questions. lxxx

.How many pounds of direct materials were purchased and used in the production of 22,500 units? A. 138,000 lbs. C. 135,000 lbs. B. 132,000 lbs. D. 137,300 lbs.

lxxxi

. What was the actual cost per pound of material? A . P3 .0 0 C. P 2. 95 B. P3.05 D. P3.10

lxxxii

. How many actual direct labor hours were worked during the period? A. 18,000 C. 19,400 B. 16,600 D. 18,970

lxxxiii

. How much actual variable manufacturing overhead cost was incurred during the period? A. P55,300 C. P56,900 B. P58,200 D. P59,500

Productivity measures Manufacturing cycle efficiency lxxxvi . Fireout Company manufactures fire hydrants in Bulacan. The following information pertains to operations during the month of  May: Processing hours (average per batch) 8.0 Inspection hours (average per batch) 1.5 Waiting hours (average per batch) 1.5 Move time (average per batch) 1.5 Units per batch 20 units  The manufacturing cycle efficiency (MCE) is: A . 72. 7% C. 6 4. 0% B. 36.0% D. 76.0%  Throughput time lxxxvii . Choco Company manufactures fire hydrants in Bulacan. The following information pertains to operations during the month of  May: Processing time (average per batch) 8.0 hours Inspection time (average per batch) 1.5 hours Waiting time (average per batch) 1.5 hours Move time (average per batch) 1.5 hours Units per batch 20 units  The throughput time is: A. 12.5 hours C. 4.5 hours B. 8.0 hours D. 9.5 hours Delivery cycle time

Standard Costs and Variance Analysis

the period is A. P5,300 unfavorable B. P1,500 unfavorable

fact, that yesterday she called you into her office and asked you to attend the executive committee meeting this morning for the purpose of leading a discussion on the variances reported for last period. Anxious to favorably impress the executive committee, you took the variances and supporting data home last night to study.

C. P1,200 unfavorable D. P6,500 unfavorable

lxxvii

. The fixed overhead budget (spending) variance for the period is A. P6,300 unfavorable C. P2,500 unfavorable B. P1,500 unfavorable D. P1,000 unfavorable

On your way to work this morning, the papers were laying on the seat of your new, red convertible. As you were crossing a bridge on the highway, a sudden gust of wind caught the papers and blew them over the edge of the bridge and into the stream below. You managed to retrieve only one page, which contains the following information:

lxxviii

. The fixed overhead volume (denominator) variance for the period is A. P 750 unfavorable C. P2,500 unfavorable B. P1,500 unfavorable D. P1,000 unfavorable

Standard Cost Summary Direct materials, 6 pounds at P3

Comprehensive lxxix . Big Marat, Inc. began operations on January 3. Standard costs were established in early January assuming a normal production volume of 160,000 units. However, Big Marat produced only 140,000 units of product and sold 100,000 units at a selling price of  P180 per unit during the year. Variable costs totaled P7,000,000, of  which 60% were manufacturing and 40% were selling. Fixed costs totaled P11,200,000, of which 50% were manufacturing and 50% were selling. Big Marat had no raw materials or work-in-process inventories at December 31. Actual input prices and quantities per unit of product were equal to standard. Using absorption costing, Big Marat’s income statement would show: A. B. C. D. C ost o f G oo ds S ol d a t P8,200,0 P7,200,0 P6,500,0 P7,000,00 Standard Cost 00 00 00 0 Overhead Volume Variance P800,000P800,000 P700,000 P700,000 U F U F

P18.0 0 4.00 2.40 5.60 P30.0 0

Direct labor, 0.8 hours at P5 Variable overhead, 0.8 hours at P3 Fixed overhead, 0.8 hours at P7

 Total Standard Cost

Direct materials Direct labor

P405,000 90,000

V A R IA N C ES R E P O RT E D Price Spendin Quantit Volume or g or y or Rate Budget Efficienc y P6,90 P9,000 0F U 4,850 7,000 U U P1,300 F ?@

Variable 54,000 overhead Fixed 126,000 500 F overhead Applied to Work in process during the period @ Figure obliterated.

Questions No. 80 through 85 are based on the following information:  You have recently graduated from a university and have accepted a position with Villar Company, the manufacturer of a popular consumer product. During your first week on the job, the vice president has been favorably impressed with your work. She has been so impressed, in

 You

recall

that

manufacturing

overhead

P14,000 U

cost

is

applied

to

219

Standard Costs and Variance Analysis

production on the basis of direct labor-hours and that all of the materials purchased during the period were used in production. Since the company uses JIT to control work flows, work in process inventories are insignificant and can be ignored.

Productivity measures Manufacturing cycle efficiency lxxxvi . Fireout Company manufactures fire hydrants in Bulacan. The following information pertains to operations during the month of  May: Processing hours (average per batch) 8.0 Inspection hours (average per batch) 1.5 Waiting hours (average per batch) 1.5 Move time (average per batch) 1.5 Units per batch 20 units  The manufacturing cycle efficiency (MCE) is: A . 72. 7% C. 6 4. 0% B. 36.0% D. 76.0%

It is now 8:30 A.M. The executive committee meeting starts in just one hour; you realize that to avoid looking like a bungling fool you must somehow generate the necessary “backup” data for the variances before the meeting begins. Without backup data it will be impossible to lead the discussion or answer any questions. lxxx

.How many pounds of direct materials were purchased and used in the production of 22,500 units? A. 138,000 lbs. C. 135,000 lbs. B. 132,000 lbs. D. 137,300 lbs.

 Throughput time lxxxvii . Choco Company manufactures fire hydrants in Bulacan. The following information pertains to operations during the month of  May: Processing time (average per batch) 8.0 hours Inspection time (average per batch) 1.5 hours Waiting time (average per batch) 1.5 hours Move time (average per batch) 1.5 hours Units per batch 20 units  The throughput time is: A. 12.5 hours C. 4.5 hours B. 8.0 hours D. 9.5 hours

lxxxi

. What was the actual cost per pound of material? A . P3 .0 0 C. P 2. 95 B. P3.05 D. P3.10

lxxxii

. How many actual direct labor hours were worked during the period? A. 18,000 C. 19,400 B. 16,600 D. 18,970

lxxxiii

. How much actual variable manufacturing overhead cost was incurred during the period? A. P55,300 C. P56,900 B. P58,200 D. P59,500

Delivery cycle time lxxxviii . Alabang Corporation is a highly automated manufacturing firm.  The vice president of finance has decided that traditional standards are inappropriate for performance measures in an automated environment. Labor is insignificant in terms of the total cost of  production and tends to be fixed, material quality is considered more important than minimizing material cost, and customer satisfaction is the number one priority. As a result, production and delivery performance measures have been chosen to evaluate performance. The following information is considered typical of the

lxxxiv

. What is the total fixed manufacturing overhead cost in the company’s flexible budget? A. P112,500 C. P139,500 B. P140,000 D. P125,500

lxxxv

. What were the denominator hours for last period? A. 18,000 hours C. 20,000 hours B. 22,000 hours D. 25,000 hours 220

Standard Costs and Variance Analysis

time involved to complete and ship orders. Waiting Time: From order being placed to start of production From start of production to completion Inspection time Processing time Move time  The Delivery Cycle Time is: A. 22 days C. 14 days B. 11 days D. 7 days lxxxix

8.0 days 7.0 days 1.5 days 3.0 days 2.5 days

. Fixed manufacturing overhead was budgeted at P500,000 and 25,000 direct labor hours were budgeted. If the fixed overhead volume variance was P12,000 favorable and the fixed overhead spending variance was P16,000 unfavorable, fixed manufacturing overhead applied must be A. P516,000 C. P512,000 B. P488,000 D. P496,000

Standard Costs and Variance Analysis

production on the basis of direct labor-hours and that all of the materials purchased during the period were used in production. Since the company uses JIT to control work flows, work in process inventories are insignificant and can be ignored.

Productivity measures Manufacturing cycle efficiency lxxxvi . Fireout Company manufactures fire hydrants in Bulacan. The following information pertains to operations during the month of  May: Processing hours (average per batch) 8.0 Inspection hours (average per batch) 1.5 Waiting hours (average per batch) 1.5 Move time (average per batch) 1.5 Units per batch 20 units  The manufacturing cycle efficiency (MCE) is: A . 72. 7% C. 6 4. 0% B. 36.0% D. 76.0%

It is now 8:30 A.M. The executive committee meeting starts in just one hour; you realize that to avoid looking like a bungling fool you must somehow generate the necessary “backup” data for the variances before the meeting begins. Without backup data it will be impossible to lead the discussion or answer any questions. lxxx

.How many pounds of direct materials were purchased and used in the production of 22,500 units? A. 138,000 lbs. C. 135,000 lbs. B. 132,000 lbs. D. 137,300 lbs.

 Throughput time lxxxvii . Choco Company manufactures fire hydrants in Bulacan. The following information pertains to operations during the month of  May: Processing time (average per batch) 8.0 hours Inspection time (average per batch) 1.5 hours Waiting time (average per batch) 1.5 hours Move time (average per batch) 1.5 hours Units per batch 20 units  The throughput time is: A. 12.5 hours C. 4.5 hours B. 8.0 hours D. 9.5 hours

lxxxi

. What was the actual cost per pound of material? A . P3 .0 0 C. P 2. 95 B. P3.05 D. P3.10

lxxxii

. How many actual direct labor hours were worked during the period? A. 18,000 C. 19,400 B. 16,600 D. 18,970

lxxxiii

. How much actual variable manufacturing overhead cost was incurred during the period? A. P55,300 C. P56,900 B. P58,200 D. P59,500

Delivery cycle time lxxxviii . Alabang Corporation is a highly automated manufacturing firm.  The vice president of finance has decided that traditional standards are inappropriate for performance measures in an automated environment. Labor is insignificant in terms of the total cost of  production and tends to be fixed, material quality is considered more important than minimizing material cost, and customer satisfaction is the number one priority. As a result, production and delivery performance measures have been chosen to evaluate performance. The following information is considered typical of the

lxxxiv

. What is the total fixed manufacturing overhead cost in the company’s flexible budget? A. P112,500 C. P139,500 B. P140,000 D. P125,500

lxxxv

. What were the denominator hours for last period? A. 18,000 hours C. 20,000 hours B. 22,000 hours D. 25,000 hours 220

Standard Costs and Variance Analysis

time involved to complete and ship orders. Waiting Time: From order being placed to start of production From start of production to completion Inspection time Processing time Move time  The Delivery Cycle Time is: A. 22 days C. 14 days B. 11 days D. 7 days

8.0 days 7.0 days 1.5 days 3.0 days 2.5 days

lxxxix

. Fixed manufacturing overhead was budgeted at P500,000 and 25,000 direct labor hours were budgeted. If the fixed overhead volume variance was P12,000 favorable and the fixed overhead spending variance was P16,000 unfavorable, fixed manufacturing overhead applied must be A. P516,000 C. P512,000 B. P488,000 D. P496,000

221

i

ii

iii

iv

.

.

.

Answer: D Nyclyn Salex Protet  Total

(24 ÷ 0.80 x P15) P 450.00 (19.20 ÷ 0.80 x P21) 504.00 (10 x P28) 280.00 P1,234.00

Answer: C Required inputs to be placed in process per unit of product: 2.25 ÷0.75 Standard Material cost per unit of product: 3.0 x P150 P450 Answer: B Required inputs of raw materials (in pounds) (60 ÷ 0.80) Standard price per pound (2.5 x 0.98) x 2.45 Standard materials cost per unit 183.75 Answer: D

3.0 yards

75.00

Standard Costs and Variance Analysis

time involved to complete and ship orders. Waiting Time: From order being placed to start of production From start of production to completion Inspection time Processing time Move time  The Delivery Cycle Time is: A. 22 days C. 14 days B. 11 days D. 7 days

8.0 days 7.0 days 1.5 days 3.0 days 2.5 days

lxxxix

. Fixed manufacturing overhead was budgeted at P500,000 and 25,000 direct labor hours were budgeted. If the fixed overhead volume variance was P12,000 favorable and the fixed overhead spending variance was P16,000 unfavorable, fixed manufacturing overhead applied must be A. P516,000 C. P512,000 B. P488,000 D. P496,000

221

i

ii

iii

iv

v

.

.

.

.

.

Answer: D Nyclyn Salex Protet  Total

(24 ÷ 0.80 x P15) P 450.00 (19.20 ÷ 0.80 x P21) 504.00 (10 x P28) 280.00 P1,234.00

Answer: C Required inputs to be placed in process per unit of product: 2.25 ÷0.75 Standard Material cost per unit of product: 3.0 x P150 P450 Answer: B Required inputs of raw materials (in pounds) (60 ÷ 0.80) Standard price per pound (2.5 x 0.98) x 2.45 Standard materials cost per unit 183.75 Answer: D Net price per yard: Purchase price 40.00 Freight 1.00 Purchase discount 0.03 x 40 ( 1.20) Standard cost per yard 39.80 Standard quantity per scarf 0.475/0.95 Standard cost per scarf: 0.50 x 39.80

.

Answer: B Weekly wages per worker 1,000 Fringe benefits (1,000 x 0.25) 250 Total weekly direct labor cost per worker 1,250 Labor cost per hour (1,250 ÷ 40 hrs) 31.25 Labor cost per unit (31.25 x 2.50 hrs)P78.125

vii

.

Answer: D Required number of quarts of berries (6 ÷ 0.80) Cost of berries (7.50 @ P8.00) 60 Cost of other ingredients (10 x 4 50) 45

75.00

0.50 19.90

Answer: C  Total Minutes per worker (8 hours x 60) Rest Time 60 Productive minutes 420 Output per day per worker (420 ÷ 35) in 10-liter batch Production hour – good units 35 min Rest minutes (60 ÷ 12) 5 min Minutes used for rejects (35 + 5) ÷ 5 good units  Total standard minutes per 10-good liter batch 48 Min

vi

3.0 yards

7.50

480

12

8 min

i

ii

iii

iv

v

.

.

.

.

.

Answer: D Nyclyn Salex Protet  Total

(24 ÷ 0.80 x P15) P 450.00 (19.20 ÷ 0.80 x P21) 504.00 (10 x P28) 280.00 P1,234.00

Answer: C Required inputs to be placed in process per unit of product: 2.25 ÷0.75 Standard Material cost per unit of product: 3.0 x P150 P450 Answer: B Required inputs of raw materials (in pounds) (60 ÷ 0.80) Standard price per pound (2.5 x 0.98) x 2.45 Standard materials cost per unit 183.75 Answer: D Net price per yard: Purchase price 40.00 Freight 1.00 Purchase discount 0.03 x 40 ( 1.20) Standard cost per yard 39.80 Standard quantity per scarf 0.475/0.95 Standard cost per scarf: 0.50 x 39.80

.

Answer: B Weekly wages per worker 1,000 Fringe benefits (1,000 x 0.25) 250 Total weekly direct labor cost per worker 1,250 Labor cost per hour (1,250 ÷ 40 hrs) 31.25 Labor cost per unit (31.25 x 2.50 hrs)P78.125

vii

.

Answer: D Required number of quarts of berries (6 ÷ 0.80) Cost of berries (7.50 @ P8.00) 60 Cost of other ingredients (10 x 4.50) 45 Standard materials cost per batch 105

viii

.

ix

.

x

.

Answer: A Actual quantity used 1,066 Add favorable quantity (209/5.5) 38 Standard quantity allowed 1,104

xi

.

Answer: C Actual materials price

75.00

0.50 19.90

Answer: C  Total Minutes per worker (8 hours x 60) Rest Time 60 Productive minutes 420 Output per day per worker (420 ÷ 35) in 10-liter batch Production hour – good units 35 min Rest minutes (60 ÷ 12) 5 min Minutes used for rejects (35 + 5) ÷ 5 good units  Total standard minutes per 10-good liter batch 48 Min

vi

3.0 yards

480

12

8 min

7.50

Answer: C Minutes required by sorting good berries 6 quarts @ 3 min. 18 Minutes required by mixing 12 Total number of minutes 30 Standard labor cost per batch (0.50 @ P50) P25 Answer: A Unit cost of materials: (Debit to Goods in Process + Debit to Materials Quantity Variance - Credit to Materials Price Variance)/Number of Units Completed  Total debits to work in process accountP51,690 Debit to materials quantity variance 1,970 Credit to materials price variance( 3,740) Actual materials cost P49,920 Per unit cost: P49,920/96,000 P0.52

105,000/35,000

3.00

Standard Quantity 12,000 x 2 24,000 Standard price 60,000/24,000 2.50 Actual Quantity used: 24,000 + (2,500/2.5) 25,000 Price variance based on usage: 25,000 x (3 – 2.50) xii

.

Answer: D Actual Quantity used Favorable Quantity Standard Quantity allowed Production in units

14,910 3,735/1.5 17,400 17,400/15

2,490 1,160

xiii

.

Answer: A AQ @ SP (3,150 x 40) P126,000 SQ @ SP (600 x 5 x 40) 120,000 Unfavorable Quantity VarianceP 6,000

xiv

.

Answer: A Actual quantity used (pounds) 23,500 Less: Excess pounds used (1,000 ÷ 2) 500 Standard Quantity Allowed 23,000 Alternative Solution using the formula for Usage Variance: MUV = (AQ –SQ)SP 1,000 = (23,500 – SQ)2 1,000 ÷ 2 = 23,500 – SQ 500 = 23,500 – SQ SQ = 23,000

xv

.

xvi

12,500

Answer: D Actual Purchase Costs – (AQ x SP) = 84,000 – (30,000 x 3) 6,000 Favorable Standard Price = Usage Variance ÷ (AQ – SQ) 3,000 ÷ (30,000 – 29,000) = P3

. Answer: B  The actual purchase price per unit can be conveniently solved by using the purchase price variance - MPV = AQ(AP-SP) -240 = 1,600 (AP – 3.60) -240 ÷ 1,600 = AP – 3.60 0.15 = AP – 3.60 AP = 3.45

xvii

.

xviii

.

Answer: MUV = = =

xix

.

Answer: C Actual materials cost 26,400 AQ @ SP (22,000 x 1.25) 27,500 Favorable Price Variance( 1,100)

xx

.

Answer: A Standard materials cost per batch (200 x 1) + (840 x 0.20) + (7 x 2) + (3 x 6) P400 Expected yield in batch (20,160 ÷ 1,050) 19.20 Actual yield 18.50 Unfavorable yield in batch 0.70 Unfavorable yield variance (0.70 x 400) P 280

xxi

.

Answer: C MPV = 1,400(1.10 – 1.00) = 140 A (AQ – SQ)SP (2,300 – 2,100) 6.25 1,250 Unfavorable

Answer: D Materials Actual cost Budgeted cost (8,500 @ 15) Materials cost variance Labor

127,500 127,500 0

AH @ SR (6,375 @ 12) SH @ SR (8,500 @ 0.75 @ 12) Labor Efficiency Variance Actual Payroll AH @ SR (6,375 @ 12) Labor Rate Variance xxii

76,500 76,500 0 77,775 76,500 1,275

.

Answer: C (AR - SR) x AH = rate variance  Therefore, the total variance (P654.50) when divided by the hourly difference (P4.27 P4.10) will equal the actual hours. Actual hours (P654.50/P.17) = 3,850. Proof: (P4.27 - P4.10) x 3,850 = P654.50

xxiii

.

xxiv

.

xxv

.

Answer: A LaborAHStd. Mix at AHDiffSRLabor Mix VarianceM4,5005,000(500)P10P (5,000)F3,0002,50050052,5007,5007,500-P (2,500)Fav Answer: A Labor Yield Variance: Expected Yield 40,000 Actual Yield 36,000 Difference 4,000 Multiply by Standard labor cost per unit P1.5625*  Yield Variance P6,250U *Standard cost ÷ Standard Yield = P6,250 ÷ 4,000 = P1.5625 Answer: C Direct labor cost at standard rate 7,150 – 750 Standard rate 6,400/800

6,400 8.00

xxvi

.

Answer: A Actual cost 10,000 Favorable Rate Variance 1,000 Actual hours @ standard rate 11,000 Standard Rate: 11,000 ÷ 2,000 5.50 Expected yield (400,000 units / 750 hrs) 7,500 = 40,000

xxvii

.

Answer: C LEV: (20,000 – 21,000)6.15 = (6,150)F Standard Rate: 3,000 = 126,000 – 20,000SR 123,000 = 20,000SR SR = 6.15

xxviii

.

Answer: C LEV: (410 x 20) – 8,440 = (240)F

xxix

.

Answer: C Actual hours 1,148,000/16.40 Less Unfavorable hours 120,000/16 Standard hours allowed Standard rate: 960,000/60,000

70,000 7,500 62,500 16.00

xxx

.

Answer: B SR = LEV ÷ (AH – SH) = -4,000 ÷ (29,000 – 30,000) = P4.00

xxxi

.

Answer: C AR = SR – (LRV ÷ AH) AR = P4.00 – (5,800 ÷ 29,000) = P3.80

xxxii

.

Answer: B Variable OH rate/hr - P27,000 ÷ 45,000 P 0.60 Direct labor rate/hr = P0.60 ÷ 0.20 P 3.00 Variable OH is applied at 20% of direct labor cost Actual hours P140,700 ÷ (P3 – P0.20) 50,250 Unfavorable hours P5,100 ÷ P3

1,700

SH allowed xxxiii

.

Answer: B Actual direct labor costs Actual hrs at std labor rate (34,500 x P6.4) Unfavorable labor rate variance Standard labor rate: -3,200 = (34,500 – 35,000) SR 3,200 = 500SR SR = 6.40

xxxiv

.

Answer: B Actual hours = Labor rate variance ÷(AR-SR) P12,000 ÷ (P10 – P9) 12,000 hours

xxxv

.

Answer: D LRV = AH(AR – SR) -5,500 = 10,000(7.50 – SR) -5,500 ÷ 10,000 = 7.5 – SR -0.55 = 7.50 – SR SR = 8.05

48,550

P241,500 220,800 P 20,700

xxxvi

. Answer: D  The controllable variance is the sum of the spending variances plus the efficiency variance. Variable overhead spending variance P( 3,600) Fixed overhead spending variance P(10,000) Variable overhead efficiency variance P 6,000  Total controllable variance P 7,600  The volume variance is not considered a controllable variance.

xxxvii

.

xxxviii

.

Answer: A Monthly budgeted fixed overhead (150,000/12) Applied fixed overhead (2,450 x 2 x 2.5)12,250 Unfavorable volume variance 250 Answer: A Variable OH per DLH 48,000/24,000 Actual overhead Budgeted OH at standard hours: Variable 21,000 x 2 42,000 Fixed 108,000 Favorable controllable/budget variance

12,500

2.00 147,000

150,000 ( 3,000)

xxxix

.

xl

.

Answer: D Actual overhead 230,000 Less Budgeted OH at standard hours Variable32,000 x 5 160,000 Fixed 64,000 (224,000) Unfavorable budget variance 6,000

xli

.

Answer: C Actual overhead 14,000 Budget at SH 15,600 Favorable controllable variance ( 1,600)

xlii

.

Answer: C OH application rate based on DL cost 600,000/(50,000 x 6) Applied overhead 325,000 x 2 650,000 Actual overhead 620,000 Overapplied Overhead 30,000

xliii

.

Answer: A Budgeted fixed overhead 81,000 Applied fixed overhead based on 80% achieved (24,000 x 3)72,000 Unfavorable volume variance 9,000 Fixed overhead rate based on 27,000 hours: (81,000 ÷ 27,000) 3.00

Answer: D

200%

Actual overhead Budget at standard hours: Fixed OH Variable OH (32,000 x 5) Unfavorable controllable variance xliv

.

230,000 64,000 160,000

Answer: B Budgeted fixed overhead Applied FOH (25,000 x 2) Unfavorable volume variance

224,000 6,000

(30,000 x 2) 50,000 10,000

60,000

xlv

.

Answer: C Efficiency variance = (AH – SH) x SVOHR (14,000 – 13,500) 6 = 3,000 UNF Standard hours: 4,500 x 3 13,500

xlvi

.

Answer: D Efficiency Variance = (31,500 – 30,000) 1015,000 Unfavorable Standard hours: 20,000 units x 1.5 hours

xlvii

.

Answer: A Fixed overhead rate per hour 8.50 – 6.00 Denominator hours (previous number) 40,000/2.5

xlviii

.

Answer: B Actual OH (10,300 + 19,500) Less: Budgeted OH at actual hours Unfavorable spending variance

2.50 16,000

P29,800 (P2 x 9,500 hrs) + P10,000 P 800

29,000

xlix

.

Answer: C EV = (AH – SH) SVOHR (14,000 – 13,500) 63,000U SH (4,500 x 3) 13,500

l

.

Answer: A Actual OH P15,000 Budgeted OH at actual hours (3,500 x P2.50) + P7,000 15,750 Favorable spending variance P( 750)

li

.

Answer: A Fixed OH spending variance: Actual Fixed OH - Budgeted Fixed OH (P315,000 – P300,000) P15,000 U Fixed OH volume variance: (Budgeted Units – Actual Units) x SFOH rate (50,000 – 55,000) x P6 P(30,000)F Budgeted production: P300,000 ÷ P3 ÷ 2 hours

lii

.

Answer: A Actual variable overhead AH @ SVOHR (270,000 x 2) Variable Oh spending variance, Favorable AH @ SVOHR SH @ SVOH (260,000 x 2) Unfavorable VOH efficiency variance

liii

liv

lv

lvi

.

.

.

.

Answer: D Actual Budget (4,500 x 2.40) Favorable Budget variance

520,000 540,000 ( 20,000) 540,000 520,000 20,000

P10,100 10,800 P( 700)

Answer: C Fixed overhead per hour: 16 x 0.7 Annual fixed OH budget 5,000 x 12 x 11.20 Answer: B Actual variable overhead 62,400 Variable OH applied 62,000 Unfavorable variable OH variance Answer: C

400

11.20 672,000

Applied fixed overhead (3,000 x 3 x 3.50) Less: Favorable volume variance Budgeted fixed overhead lvii

.

Answer: A Standard rate: Excess rate Actual rate

50,000/40,000 1.25 1,080/3,600 0.03 1.28

lviii

.

lix

.

lx

.

Answer: A Net OH variance, Unfavorable 2,000 Less: Unfavorable volume variance( 18,000) Unfavorable spending variance( 25,000) Favorable FOH budget variance 41,000

lxi

.

Answer: C Budgeted fixed OH 500,000 Add: Favorable volume variance 12,000 Applied fixed overhead 512,000

lxii

.

Answer: A Applied FOH (8,000 x 6) 48,000 Less: Favorable volume variance Budgeted FOH 36,000

lxiii

lxiv

.

.

31,500 875 30,625

Answer: A Applied fixed overhead 48,000 Less favorable volume variance12,000 Budgeted fixed overhead 36,000 Answer: A Unfavorable volume variance 25,000 Unfavorable VOH spending variance18,000  Total 43,000 Net Unfavorable variance 2,000 Favorable fixed OH budget variance41,000

12,000

Answer: A Budgeted fixed OH (3,000,000 ÷ 12 months) Applied fixed OH (26,000 @ 2 x 5) 260,000 Favorable volume variance ( 10,000)F Fixed OH rate per hour (3,000,000 ÷ 600,000) Answer: B Labor Efficiency: (53,500 – 52,000) 812,000 Variable OH Efficiency (53,500 – 52,000) 6 Total efficiency variance 21,000

250,000

5.00

9,000

lxv

.

lxvi

. Answer: D Fixed overhead volume variance is a more meaningful variance in evaluating the use of the capacity.

lxvii

.

lxviii

lxix

.

.

Answer: D  Total variable overhead variance (80,000 – 90,000)10,000 favorable Variable overhead spending variance 1,200 favorable Variable overhead efficiency variance 8,800 favorable 8,800 ÷ 20 440 Favorable

Answer: B Actual variable OH Budgeted VOH at actual hours (80,000 x P3) Unfavorable VOH spending variance Answer: A (38,000 units – 50,000 units) x P8

P96,000

Answer: B Spending [P315,000 – (53,500 x P6)]P(6,000)

P250,000 240,000 P 10,000

Efficiency [(53,500 – 52,000) x P6]P 9,000 lxx

.

Answer: B Spending [P260,000 – (P3M ÷ 12)]P10,000U Volume [(26,000 – 25,000) x P10]P10,000F

lxxi

.

Answer: C Actual fixed overhead P88,000 Budget fixed overhead (4,000 hrs @ P20) 80,000 Unfavorable fixed OH Spending variance P 8,000 Budgeted (denominator) hours (40,000 units x 6 ÷ 60)

lxxii

.

Answer: B Budget fixed overhead P80,000 Applied fixed overhead (38,000 x 0.10 x P20) Unfavorable volume variance P 4,000

4,000

76,000

lxxiii

.

Answer: A Actual variable overhead P 16,400 Budget at actual hours (4,200 x P4) 16,800 Favorable variable OH spending variance P ( 400)

lxxiv

.

Answer: C Unfavorable Efficiency Variance: (AH – SH) SVOHR (4,200 – 3,800) x P4 = 1,600 UNF SH allowed (38,000 units x 1 ÷ 10) = 3,800 hours

lxxv

.

Answer: A Actual variable overhead 108,500 Budgeted VOH at actual hours (17,200 x 6) 103,200 Variable overhead spending variance, UNF 5,300 VOH rate per hour (135,000 x 0.80) ÷ 18,000 hours P6.00

lxxvi

. Answer: C  The computation of variable overhead efficiency variance involves the comparison of  the actual hours and standard hours allowed by actual production. (17,200 – 17,000) x P6 1,200 UNF Standard hours allowed: 8,500 x 2 17,000

lxxvii

. Answer: D  The amount of fixed overhead budget (spending) variance is calculated by subtracting from the actual fixed overhead the amount of budgeted fixed overhead. Actual fixed overhead 28,000 Budgeted fixed overhead (135,000 x 0.2) 27,000 Unfavorable fixed overhead budget variance 1,000 . Answer: B  The amount of volume variance (denominator or over/underapplied fixed overhead variance) is calculated by comparing the budgeted fixed overhead and fixed overhead applied to production. Budgeted fixed overhead (135,000 x 0.2) 27,000 Applied fixed overhead (8,500 x 3) 25,500 Underapplied (unfavorable) volume variance 1,500 Alternative calculation: (9,000 – 8,500) x 3 1,500 Fixed overhead per unit (27,000 ÷ 9,000) 3

lxxviii

lxxix

lxxx

.

.

Answer: C Std unit cost: Variable (7,000,000 x 0.60) ÷ 140,000 P30 Fixed OH (11,200,000 x 0.50) ÷ 160,000 35 Std unit cost P65 CGS – Std (100,000 x 65) 6,500,000 OH Volume Variance: (160,000 – 140,000) x 35 Answer: A SQ allowed (22,500 x 6) Unfavorable usage variance Actual quantity of materials

135,000 3,000 138,000

P 700,000 UNF

lxxxi

lxxxii

.

Answer: C Actual quantity purchased and used at standard price (138,000 x 3) Favorable price variance 6,900 Actual Quantity @ Actual Price 407,100 Actual Price (407,100 ÷ 138,000) P2.95

.

lxxxiii

.

lxxxiv

.

lxxxv

90,000 7,000 97,000 19,400

Answer: D AH @ SR (19,400 x 3) Spending variance Actual Variable Overhead

58,200 1,300 59,500

Answer: B Applied Fixed OH Underapplied fixed overhead Budgeted fixed overhead

.

126,000 14,000 140,000

Answer: C Denominator or Budgeted Hours: (140,000 ÷ 7) = 20,000

lxxxvi

.

Answer: C MCE = Value Added Hours ÷ Throughput Time Processing hours 8.00 Inspection hours 1.50 Waiting time 1.50 Move time 1.50  Throughput time 12.50 MCE (8.00 ÷ 12.50) 64%

lxxxvii

.

lxxxviii

.

lxxxix

Answer: C SH @ SR Efficiency Variance AH @ SR Actual hours (97,000 ÷ 5)

414,000

Answer: A Answer: A Delivery cycle time:  Total waiting time Inspection time Processing time Move time Delivery Cycle Time

15.00 1.50 3.00 2.50 22.00

. Answer: C A favorable volume variance arises when the applied fixed overhead is higher than the budgeted fixed overhead. Budgeted fixed overhead 500,000 Favorable volume variance (overapplied) 12,000 Applied fixed overhead 512,000

View more...

Comments

Copyright ©2017 KUPDF Inc.
SUPPORT KUPDF